• Practice Test
  • Useful Tips – Tricks
  • Full Writing Review
  • General Writing Task
  • Writing Task 1
  • Writing Task 2
  • Writing Exercises
  • Writing Sample – Topics
  • Writing Vocabulary
  • Speaking Vocabulary
  • Intro Question
  • Speaking Part 1
  • Speaking Part 2
  • Speaking Part 2 – Audio
  • Speaking Part 3
  • IELTS Books
  • Recent Exams
  • IELTS Vocabulary
  • Essay from Examiners
  • IELTS Ideas

Logo

IELTS Writing Task 2 : Art (Opinion essay)

Task 02: art.

Some people say that art (e.g. painting, music, poetry) can be made by everyone whereas others believe that it can be only made by those with special abilities. Discuss both views and give your opinion.

Sample Answer:

While some people believe that everyone has the ability to create art , others believe that only those with special talents are able. Personally, I believe that everyone has the ability to create art, however some people are more naturally gifted than others.

On one hand, creating art is something that everyone can do. However, not every painting will be a masterpiece, nor every song a hit. Whether it is a painting, a song, or a poem, art is the personal expression of the artist, and its quality is subjective to the viewer or listener. For example, while there are many people who believe that the Mona Lisa is a brilliant work of art, there are also many people who do not appreciate it. So, while people may argue over how good a painting or a song is, I believe the main factor that defines whether something is art or not is whether it is a unique expression of the individual who created it, and not about how beautiful or popular it is, and therefore, everyone can create art.

However, creating art may be far easier or enjoyable for those who are naturally gifted or talented, and the art they produce may be appreciated much more by others. Art is about expressing an idea, emotion, or feeling via a specific medium , such as painting or music, and it is much easier for those who are skilled with a paintbrush, a guitar, or the use of language. There are many people who simply become too frustrated and quickly give up because they do not have the skill or patience to be able to express what is inside of them, while those blessed with certain skills are able to express their ideas with ease and grace.

In conclusion, although I believe that everyone has the ability to create art, many people who do not have the natural talent or ability are easily frustrated and therefore give up quickly. However, this does not mean that they cannot create art, they simply lack patience and humility.

(341 words)

  • writing 2021
  • writing task 2

LATEST POSTS

Writing task 2: animal species become extinct, writing task 2: the government should spend money putting more works of art, writing task 2: benefit on a ban on all forms of advertising.

IELTS App

IELTS App - For Mobile

Ready for the IELTS exam with our IELTS app. Over 2 million downloads

Download App

Popular Last 24h

Describe a film that made you laugh, describe something difficult you would like to succeed in doing, [ebook] ielts share: 240 speaking topics pdf, list of top 100 synonyms in the ielts test, ielts speaking part 1 : hometown, village, living place, city, tourism, country, [pdf + audio] download full set of 4000 essential english words, ielts preparation tips | top-10 online free sites.

  • IELTS Test/Skills FAQs
  • IELTS Scoring in Detail
  • Forecast Speaking – 2023
  • List IELTS Speaking Part 3
  • List IELTS Speaking Part 1
  • IELTS Writing 2023 – Actual Test

Our Telegram

Join our community for IELTS preparation and share and download materials.

The information on this site is for informational purposes only. IELTS is a registered trademark of the University of Cambridge ESOL, the British Council, and IDP Education Australia. This site and its owners are not affiliated, approved or endorsed by University of Cambridge ESOL, the British Council, or IDP Education Australia.

Latest Articles

Ielts speaking part 3: topic relax, describe a place | where you go to relax, ielts speaking part 1: advertisements (audio), describe a place where you like to go shopping  , describe an event you attended, most popular, describe a person whom you met for the first time and made you happy, topic: experience is the best teacher, in many countries,today there are many highly qualified graduates without employment..

ieltspracticeonline All Rights Reserved

Band 9 Essay: Creative artist should always…

ielts essay on art band 9

Band 9 Essay: Creative artist should always be given the freedom to express their own ideas in words, pictures, music or film in whichever way they wish. There should be no government restrictions on what they do.

To what extent do you agree or disagree with this option.

Art and creativity impact our society. Individuals with creativity must always be provided the freedom of expression and beliefs (perspectives) in words, images, songs or movies as per their wish. There ought to be no restriction of authority on their works. This essay partially agrees with the above notion. Because every individual has different understanding as per their age. In the upcoming paragraphs, the positive role of creativity will be enlightened in relation to adults whereas the bad impacts it can have on children will be further elaborated.

Talking about the positive role of creativity, freedom of expression can provide better understanding to individuals. Directors, producers and even writers can feel motivated when there will be no restriction to them. Therefore, this will lead to development of good and creative content. Moreover, it will be a great source of entertainment for a mature adult. To exemplify, best of the movies are made in Hollywood, USA and it started after the restrictions were ended in 1948 by President Ronald Regan, as he ended the censorship.

IELTS Writing Module: Online Free Classes for Task 2- Click Here!!!

But, this complete sense of freedom and the ideas originated out of it can hammer the development of a child. Heavy action movies which might be source of entertainment for adults can misguide the mindset of young ones. They can copy the stunts and hurt themselves. Here, the role of censorship is very important. Movies must be provided certificates to categorize them. So that children can watch those mature movies as well but after they reach certain age. As per data of a report published by United Nations, Uganda has provided complete freedom of expression at the level of making but they certify the movies at the level of viewers. Thus, creating a balance between the both.

Concluding the above, it is true that best of the creative works can only be done in complete freedom but there has to be some government restriction as well so that content can reach the exact consumer for which it is made.

‘Creative artist should always be given the freedom to express their own ideas in words, pictures, music or film in whichever way they wish’

IELTS Writing Module: Online Free Classes For Task 1- Click Here!!

 ielts essays sample answers.

  • Happiness is considered very important in life
  • In the 20th century, contact between many different parts of the world has developed rapidly through air travel and telecommunications. Do the advantages of this outweigh the disadvantages?
  • Individual greed and selfishness have been…
  • Doing an enjoyable activity with a child can…

Leave a Reply Cancel reply

Your email address will not be published. Required fields are marked *

Save my name, email, and website in this browser for the next time I comment.

You cannot copy content of this page

Insert/edit link

Enter the destination URL

Or link to existing content

IELTS Band 9 Writing Samples: Task 2 Essays

June 19, 2021

ielts essay on art band 9

One of the best ways of learning how to write better is to simply read sample IELTS band 9 essay answers, and that is exactly what we have here: 10, Band 9 sample IELTS essays. Each essay is followed by a teaching point to show you why it is a band 9 IELTS essay.

Finally, all of the essays on this page have been written using the system I teach on this page IELTS writing task 2 and in my full IELTS course here that has helped thousands get the score they need.

You can also download these sample answers as a pdf file here if you prefer: IELTS Essay Samples Band 9 pdf or, simply read them below:

Sample Essay #1 – Two Part Question

In some countries, the number of people visiting art galleries is reducing. What do you think the reasons for this are? How can we solve this problem?

In certain locations around the world, the number of people visiting art galleries is declining. This essay shall outline some of the reasons for this trend and then go on to suggest ways in which this issue could be resolved.

Firstly, visitor numbers are on the decline due in part to the ever-increasing convenience and ability of new technology. If someone has access to the internet from a device then there is virtually no need to visit an art gallery as all the finest works can be viewed online for as long as you want and at a minimal cost. For example, there is virtually no reason to go to the effort of leaving your house and traveling across a city and then paying and queuing with other people just to see works of art that you could just as easily view from the comfort of your own home.

However, there are some effective ways in which we might reverse the trend of declining visitor numbers to art galleries. One such way would be to ensure that all the artwork at a gallery is not available to view online, or at the most, just a small sample of an art galleries work is available for viewing. This would then create a sense of curiosity in the viewers mind and make them more likely to visit the art gallery. Furthermore, you could create a discussion zone at the art gallery where like-minded individuals could meet face to face and discuss the particular pieces of art that interest them. This would make visiting the gallery a more unique experience and be more likely to catch people’s interest.

Overall, visitor numbers are declining but there are a number of ways to tackle this problem. It is up to the art galleries themselves to come up with solutions and then deliver these to the public if they wish to survive in the future. 319 words

Teaching Point: Notice how both of these topic sentences directly answer one of the questions asked in the question. This is key to making sure that you do not go off topic and do in fact answer the question. This ensures you will not lose marks for Task Achievement.

Sample Essay #2 – Discussion And Opinion

In many countries, men and women work full-time. It is therefore logical for men and women to share household work. To what extent do you agree or disagree?

Nowadays, many people believe that men and women should share household chores equally as both genders are just as likely to have full-time jobs. Personally, I agree with this viewpoint and the following paragraphs shall outline my reasons for this belief.

First and foremost, traditional gender roles have been severely diminished in many cultures in recent years. This means that less pressure is now placed on women to complete the tasks that were commonly associated as being a women’s job to complete. These days it is just as acceptable for a man to do the housework as it is for the women, and they won’t be looked down upon by their male friends as they might have been in the past.

Secondly, it has become much more commonplace for women to be the main breadwinners of a household and therefore by default have less time available for domestic duties This means that it often makes more sense for men to stay at home and not work, which in turn means that they have more time available to complete household chores than might have been the case in the past. Imagine, if a woman worked full time and then had to come home and complete all of the household chores as well, regardless of whether the partner was working or not, the relationship would be put under a great deal of pressure and might eventually end if they were left to do the chores alone.

In summary, I agree that the changing trends of society mean that couples are often led to divide household chores more equally these days. Despite resistance by certain groups, this trend is likely to continue into the future.

Teaching Point: Notice how I have repeated my opinion twice, in both the introduction and conclusion but have done so using different words. This shows off a range of vocabulary but also ensures that I have answered the original question that was asked.

Sample Essay #3 – Discussion And Opinion

Libraries are a waste of money, therefore, computers should be used to replace them. Discuss both views and give your own opinion.

Some people are of the opinion that libraries funding should be cut and the money invested in making computers available to the public instead. I mostly agree with this line of thought and the following paragraphs shall explain why this is the case.

Firstly, libraries should not receive any more funding because they contain such a limited and often outdated amount of information. As soon as a book is published it goes out of date and cannot be updated without an entirely new copy being printed which is both costly and time-consuming. On the other hand, a computer connected to the internet overcomes both of these limitations with ease, for example, any web-site, pdf, or online journal can be continuously updated by the authors and there is no time wasted in printing of the book.

On the other hand, though, libraries do still offer a quiet place for members of the public to go and read. In today’s fastpaced society there are few places to be found where people can simply go and relax without fear of being hassled by salesman or traffic which may have damaging consequences for the public. For instance, a report in the ‘Journal of Good Health’ recently reported that spending as little as 5 minutes per day sat quietly on your own can reduce the risk of a heart attack or stroke by 50%, so, losing the quiet space of a library could harm a nation’s overall health.

To sum up, the public need for up to date information and also for restful places for people to relax needs to be considered carefully. Governments need to decide what their priority is and act accordingly. 279 words

Teaching Point: Notice how in the first line of the introduction I have simply paraphrased the question statement using my own words. I have also changed the order of the information in the sentence. This shows the examiner that I have good grammatical control and also a range of vocabulary.

Sample Essay #4 – Discussion And Opinion

Some people think that money is the best gift to give a teenager, others disagree. Discuss both sides and give your own opinion.

Certain groups of people are of the opinion that giving teenagers cash is the most appropriate present to present them with, however, some people disagree with this approach. Personally, I believe that this is not the case and this essay shall outline arguments for either side.

Firstly, teenagers are often very impulsive by nature and are likely to make decisions that may not be in their best overall interest. As a consequence, if you hand over money to a teenager they may well simply go and waste the money on consumable goods and sometimes harmful items such as drugs, alcohol, or other such substances. Therefore, it is probably in the teenagers best interest if you buy them constructive presents that they can get greater value and education out of. For example, buying a teenager book tokens to further their knowledge is far more productive than giving them cash to blow on alcoholic beverages.

On the other hand, however, some people would say that allowing the teenager the freedom to choose what they want to spend their money on is an important lesson for them to learn. Not only does it allow them to see that you trust them but it also means that they are likely to buy something that they will actually value. Furthermore, if a teenager senses that you do not trust them then they are likely to hold this against you and use it against you at some point in the future, whereas, they may well act more responsibly if you hand over cash for them to spend.

In conclusion, teenagers are at a very sensitive stage of their development, however, I remain of the opinion that they do need some guidance in the way that they spend their money. Parents should take care to manage this situation appropriately. 302 words

Teaching Point: Notice how the conclusion starts by summarizing the two topic sentences using different vocabulary, Once again this proves to the examiner that you have a good range of vocabulary.

Sample Essay #5 – Opinion Essay

Some people believe that people who do physical work should be paid the same as people who have a high-level degree. Do you agree or disagree?

Certain groups of people are of the opinion that people who engage in manual labour should receive the same level salary as someone who is highly educated. I disagree with this point entirely and shall outline the reasons for this in the following paragraphs.

One of the main reasons why highly educated people should receive a greater salary than lower-skilled workers is that they create more value for a business in the long term. This is because a lower skilled worker will simply do as they are told and perform their role in the organisation whereas a highly skilled worker is more likely to suggest solutions to problems or invent more productive ways of doing something. Over the course of a number of years, these incremental improvements could lead to large increases in profit for the company.

Allied to this, graduates have often invested a great deal of time and money into their education and so surely, therefore, deserve to be paid more to cover this. For example, a recent survey from ‘Time’ magazine revealed that the average medical student seeking to become a doctor graduates with more than $150,000 of debt before they have even earned a penny.

Furthermore, countries need educated populations in order to develop, organise themselves and grow. Therefore governments need to make sure students are encouraged to study for higher qualifications and paying higher salaries to these individuals when they finally graduate is one way of ensuring this.

In conclusion, more highly educated employees are worth more to a company and a country. These are the main reasons why I continue to believe they should be paid more. 273 words

Teaching Point: It is helpful to develop your paragraphs by using examples. However, this is difficult to do as you do not know what question you will be asked. This is why you should just make up realistic sounding examples. It really is not important if the example is true or not, the examiners do not care. All they want to do is assess your English. So, go ahead and simply make up realistic sounding examples to develop your answers just as I have done here!

Sample Essay #6 – Opinion Essay

In some countries, children under sixteen are not allowed to leave school by law and get full-time work. Is this a good thing or a bad thing?

In certain areas of the world, children under the age of 16 are prevented from gaining full-time employment by law. I believe this is a good thing and this essay shall outline the reasons for this standpoint.

Firstly, anyone under the age of 16 should be pursuing education rather than a salary. This is because they have the rest of their working lives to get a full-time job but only a few limited years during their youth which they can dedicate entirely to education. Education is the key to a positive future and so it is right that laws should prevent someone from damaging their own education. If we let young people simply do what they want with no thought for the future then we would not be guiding and protecting them as a society surely should.

In addition to the above, many countries around the world have high unemployment levels. If youth under the age of 16 were also added to the working population then this would likely only lead to further increases in unemployment. For example, in Greece the ‘Greek Echo’ recently reported that unemployment had increased to a record level of 38% of the population. Furthermore, having unemployed youngsters on the streets often leads to increased crime rates, especially those relating to anti-social behaviour whereas if the youngsters had to remain in school or college they may well stay out of trouble.

Overall, beginning employment early has more negative impacts than positive. Governments should consider carefully when and how they allow people to finish their education if they wish their nations to be prosperous in the future. 269 words

Teaching Point: Notice how I have used two conditional sentences here to discuss future changes. This demonstrates a wider range of grammar to the examiner and therefore helps to improve your band score. Make sure you brush up on the second conditional in particular, as it is often useful in IELTS essays.

Sample Essay # 7 – Two Part Question

Nowadays, some parents pressure their children to be successful. What are the reasons for this? Is this a positive or negative development?

In recent years, some children have been put under pressure by their parents to be successful in life. This essay shall discuss both the reasons why this is so and whether this is a positive or negative development.

It appears that some of the youth of today are placed under pressure by their parents to be successful because the world has become a very materialistic place and in order to show how successful you are you need to have money to buy nice things. This usually means that a good education is needed so a well-paying job can be secured. Unfortunately, as a consequence of students studying to gain a high paying job, which their parents may wish for them, they may actually be doing something which is not what they want to do deep down in their soul. As a result, a student may begin to lack motivation in their studies, lack of passion for what they are doing or even become depressed as a result.

Furthermore, the pressure placed on young people to succeed at school may well mean that they do not take part in other valuable opportunities. For example, rather than taking part in an International Award programme they may well opt to do extra homework because of the time required to gain the award. However, participating in the award would have provided them with so many opportunities to learn new and different life skills, such as: social skills, trip planning, map reading, fund raising, teamwork and so on, skills which you simply cannot ever learn from a book.

To sum up, anything that could cause depression or reduce a young person’s opportunities has to be a negative. Parents need to think carefully about what type of life they want their child to actually have and not just on future financial prospects. 308 words

Teaching Point : Notice that I have used a range of sentence starters and connectives to help the essay flow. I have not simply repeated the same linking words like ‘and also’, ‘then’, or ‘next’ that are often overused in IELTS essays. Using a range like this means that the essay sounds more natural and native like and of course helps improve a band score.

Sample Essay #8 – Problem And Solution

In many countries, people have health problems because they choose to live an unhealthy lifestyle. What do you think the reasons for this are and how can it be solved? Give relevant examples from your experience?

In many places around the world, people are choosing to live an unhealthy lifestyle and are suffering significant health issues as a result. The following paragraphs shall discuss the possible cause of this and offer a number of solutions.

Firstly, one of the main causes of these health issues is the influence of advertising from big businesses trying to make a profit. These businesses have no morals and are only interested in making money, this means that they will target anyone they can even though they know that their products are bad for people’s health. For example, MacDonald’s are certainly aware that their food is bad for children but they still target them through the use of associating clowns and Disney characters with their ‘happy meals’.

Allied to the above, people are ill disciplined even when it comes to the importance of their own health. These days, everyone knows the risks of eating ‘junk’ food on a regular basis but many continue to do so. The reason for this is that it is just too convenient and they are just too lazy to make some real nutritious food for themselves. For instance, anyone who goes out on a weekend will have witnessed the large queues of young people in fast food restaurants even when there are much more healthy options nearby including various supermarkets which all sell healthy ingredients from which to make food from.

In summary, the power of big business and the weak will of humans is damaging the health of many. Governments, schools and parents should consider carefully how they are going to tackle these issues in the coming years. 273 words

Teaching Point: Usually the second or third sentence of a paragraph will be explaining the reasons for what has been stated in the topic sentence of the paragraph.

Sample Essay #9 – Discussion And Opinion

Nowadays, many families move to different countries. Some people think that children gain many benefits from this while others consider it to be hard for a child to move to a foreign country. Discuss both views and give your own opinion.

These days it is not uncommon for whole families to migrate to other parts of the world. Some people are of the opinion that this has a negative impact on the children involved whereas others believe it has a positive impact. Personally, I think the positives outweigh the negatives and this essay shall outline both sides of the debate.

First and foremost, generally people only move to other countries if they believe there is going to be a significant improvement to their children’s lives. Often this takes the form of improved education opportunities. For example, when the UK entered the European Union there was an immediate influx of people and part of the reason for this is that the UK offers a free and a quality education to any youngsters living there. Many migrants believe that the key to future success is education and that moving to the UK will enhance their children’s future.

On the other hand, removing a child from the culture they have grown up in may severely disrupt their behaviour especially if they did not want to move in the first place. Teenagers and even younger children are very sensitive to change and a major change such as moving to another country could cause a lack of confidence. For example, suddenly a child has to east food they are not used to and may not like, suddenly they may also have to get used to weather they may not have even experienced before. All of these things could cause a child to experience mental health issues.

Overall, children often gain more opportunities by migrating abroad although they will face new challenges. Parents should carefully consider the potential impact a sudden move may have on a child before they make the final decision. 296 words

Teaching Point : 4 main paragraphs is usually enough for most IELTS essays. An introduction of about 50 words, two body paragraphs of about 90 words each, and a conclusion of about 30 words.

Sample Essay #10 – Discussion

Earlier technological developments brought more benefits and changed the lives of ordinary people more than recent technological developments. To what extent do you agree or disagree?

Improvements in technology that occurred in the distant past produced more positive effects than the developments that have occurred in the last few years. I completely agree with this statement and the following paragraphs shall outline the reasons for this belief.

The first telephone completely revolutionized the way business was done and benefited humanity greatly. For the first time people could send messages long distances with ease and the pace of business increased dramatically making more people richer and creating more job opportunities. However, these days when the latest iPhone update comes out the only real changes are to do with fashion rather than providing any real new benefits. For example, now you can upload items to a ‘cloud’, or play more advanced games, but neither of these improvements in anyway compares to the first time phones were released to the public.

Similarly to the above, the first computers also transformed the way companies ran their businesses. Previously there were filing cabinets full of paperwork and accessing that information could take hours to locate the piece of information that you wanted. In contrast though, computers have been around for so long now that they have almost reached their limit in terms of how useful they could possibly be. For example, the only real changes that happen now are new releases of the Windows operating system and the so called improvements are actually just considered annoying changes to many rather than actual improvements.

In summary, the most profound long lasting impacts that technology has brought us occurred many years ago. These days’ beneficial changes now come in very small increments and I believe that will continue to be the case in the future. 282 words

Teaching Point: Always start with an introduction which rephrases the question. You should try to use different words i.e. synonyms and paraphrases of the original words in the question so that you can show to the examiner your range and level of vocabulary.

Sample Essay #11 – Discussion

Nowadays, people of all ages from certain parts of the world spend most of the time at home rather than going outdoors. Discuss the reasons for this and say whether it is a positive or negative development.

In this day and age it is far more common for individuals to spend the majority of their time inside rather than outside. I believe this is a negative development for society and the following paragraphs shall offer possible reasons why this could be the case.

Firstly, spending more time indoors naturally indicates decreased activity levels. This automatically leads to reduced health of populations due to problems such as obesity, stroke, heart attack and so on which are all linked with decreased levels of exercise. Clearly this is a major negative for everyone concerned. Governments have higher health costs, people die younger and families of course are deprived of a family member needlessly.

Secondly, the fact that people are indoors more often than not indicates that less time is spent socialising with others face to face. This can lead to mental health problems but also to a decline in the development of ‘real world’ social skills which help to make people employable. Afterall, in most places of work there is a definite need to communicate face to face with colleagues of customers and if an individual is not capable of doing this it does not matter how ‘book smart’ they are they will not be able to function adequately in the workplace.

Overall, it is clear to me that there are far more negatives to positives of people spending more time indoors than outside. Governments, education authorities and parents around the world should carefully consider the consequences of such a trend. 251 words

Teaching Point: Your main body paragraphs, which are the two paragraphs in the middle of your essay, should begin with a topic sentence. This topic sentence should say what the main point of your paragraph is and does not have to be too long or complex. The reader should be able to guess what the rest of the paragraph is going to be about just from reading your topic sentence.

IELTS Essay Samples Band 9 PDF

For convenience you can also download these sample band 9 answers as a pdf file here:

IELTS Essay Samples Band 9 pdf

What To Do Next?

If you want to see the exact process I use to write essays like the above band 9 answers then there are two options. You can read my main guide to writing IELTS essays here , or you can go straight to my IELTS course page which thousands of people have used to master each part of the IELTS test.

Recommended IELTS Study Tools

Thank you for reading this article. I always get lots of questions about how else to get a better band score quickly. So, this is what I recommend:

Complete IELTS Course : Of course, my full course ‘ INCREASE YOUR IELTS ‘ covers everything you need to need to know to pass IELTS, including practice questions, model answers, grammar work, strategies for every possible reading, writing and listening question type, as well as a complete speaking course too, check it out here .

IELTS Essay and Speaking Feedback : To complete full mock tests and get feedback from IELTS examiners on your IELTS essays or speaking tasks then visit: IELTS Feedback and Mock Tests, here.

Improve your grammar fast by using the Grammarly suggestions to improve your writing. Every IELTS students should have this free grammar improving tool.

Improve all-round English skill with EnglishClass101.com . If you have failed IELTS more than once then you probably need to improve your general level of English. Use the free online lessons and vocabulary building tools here and start improving today! HIGHLY RECOMMENDED!

ielts essay on art band 9

Hi there, I’m Tim James a former IELTS examiner and IELTS teacher of over a decade. This site is where I share my exam strategies to help people get the score they need. I hope it helps you!

In A Hurry To Pass IELTS? Then Check Out…

ielts essay on art band 9

Copyright & copy; 2017 · IELTSfreeway.com · All Rights Reserved

Powered by OptimizePress 2.0

IELTS MATT

Reach your dream band score for IELTS Speaking and Writing

IELTS Writing Task 2 Problem/Solution Essay Band 9 Model Answer-Art Galleries

Here is an example showing how to write an IELTS problem/solution essay. There are two logical structures for the body paragraphs.  Option 1) Body Paragraph 1=Problem 1 and Problem 2    Body Paragraph 2=Solution 1 and Solution 2 Option 2) Body Paragraph 1=Problem 1 and Solution 1    Body Paragraph 2=Problem 2 and Solution 2

Personally, I prefer the second option because it allows the problems and solutions to be linked together more effectively. If you are having issues with any type of IELTS essay, please don’t hesitate to contact me 🙂 

In many countries the number of visitors to art galleries has been steadily declining. What are the reasons for this? What could be done to improve the situation? 

Art galleries are visited a lot less frequently than they used to be in several countries. This essay will highlight possible causes for this and will suggest some measures to halt and possibly reverse the decline.

The most obvious reason why people have stopped visiting galleries is that they seem boring in comparison to new technology. Previously, many people searched for cultural events to occupy their time, however, nowadays it can be easier and more enjoyable to spend time online. What is more, many famous artworks are available to see in high definition from the comfort of your own home. To solve this problem, galleries should create exhibits which use technology to demonstrate the artwork in a more exciting and social-media friendly manner. If curators implemented this strategy, visitors would probably share their visit on social-networks which would further promote the gallery. 

Another aspect that has reduced the popularity of art exhibitions is the escalating cost which has made museums unaffordable, especially for poorer families. When faced with the decision of cheap entertainment or visiting an art gallery, it is understandable that many families choose the more cost-effective option. To illustrate this point, Le Louvre, which is one of the most famous galleries in the world, charges 15 Euros per person, which would cost around 60 Euros for a family of four. Whereas, this same family could visit the Eiffel Tower for free. To resolve this situation, art galleries could subsidise entry for families or even provide free entry during quieter times. Admittedly, they may lose money on entrance fees, nevertheless, this may be offset by increased revenue in the gift shop or cafe.

To sum up, there are a variety of options for how we can spend our free time nowadays which are cheaper and more interesting than visiting art galleries. It seems possible that making them more affordable and embracing technology to enhance the experience would encourage more people to visit. 

guest

The aim of IELTS writing is not to use the most complicated vocabulary possible. You should consider collocations, structure and the message that you are trying to get across. Native speakers use a combination of simple and complicated words in a natural way and you should aim for the same.

David

why do u use simple words like these: suggest,solve, understandable, choose

Speak to me on Whatsapp

IELTS Luminary - Free Tips, Strategies, eBooks and Detailed Essay Feedback . Overall, this is the best free IELTS and other test prep website.

  • IELTS Luminary
  • Sep 17, 2022

Art and music are considered some of the fundamental elements of all societies.(Task 2 Band 9 Essay)

Updated: Mar 14

You should spend about 40 minutes on this task.

Write about the following topic:

Art and music are considered some of the fundamental elements of all societies.

Do you think art and music still have a place in today’s modern world of technology?

Should children spend more time learning art and music at school?

Give reasons for your answer and include any relevant examples from your own knowledge and experiences.

You should write at least 250 words.

Task 2 Band 9 Sample Essay - Art and music are considered some of the fundamental elements of all societies.

Get your personalised   IELTS Essay Feedback   from a former examiner

Download IELTS eBooks ,   get everything you need to achieve a high band score

Sample Essay 1

In an era dominated by rapid technological advancements, the relevance of art and music in shaping societies remains undiminished. These cultural cornerstones not only foster creativity but also serve as a bridge connecting diverse communities. This essay posits that the integration of art and music into the educational curriculum is imperative, not only for cultivating a well-rounded intellect but also for ensuring the holistic development of children.

Firstly, the infusion of art and music into education ignites creativity and innovation among students. These disciplines encourage unconventional thinking and problem-solving skills, attributes that are invaluable in today's fast-paced, technology-driven world. For instance, learning music theory can enhance mathematical abilities, as both require a deep understanding of patterns and structures. Similarly, engaging in art can improve spatial reasoning and fine motor skills. Such interdisciplinary benefits underscore the importance of these subjects in nurturing versatile individuals capable of contributing to various fields, including science and technology.

Moreover, art and music play a crucial role in emotional and social development. Through these mediums, children learn to express themselves more effectively, fostering emotional intelligence and empathy. Participating in group performances or art projects also enhances teamwork and communication skills, preparing students for collaborative environments in their future careers. Furthermore, exposure to the rich tapestry of global cultures through art and music cultivates open-mindedness and cultural sensitivity, traits increasingly vital in our interconnected world.

In conclusion, incorporating art and music into educational programs is crucial, not a luxury. These disciplines are key to developing creative, emotionally intelligent individuals who can adeptly navigate today's challenges. They enhance traditional academic skills, preparing students for a dynamic world. Thus, it's vital to allocate more time to these subjects, ensuring they play a significant role in nurturing future generations.

Sample Essay 2

In the digital epoch, the relevance of art and music often sparks debate, yet their role in enriching societies is indisputable. These disciplines, far from obsolete, are crucial in today's tech-savvy world. This essay will argue that not only do art and music retain their importance, but their integration into the educational syllabus is also vital for developing multifaceted, innovative individuals, thereby necessitating more dedicated learning time in schools.

Art and music education serves as a catalyst for intellectual and creative growth, vital in today's innovation-driven era. Engaging with these fields not only stimulates critical thinking but also fosters a mindset open to innovation, crucial for addressing today’s complex challenges. For instance, the abstract thinking required for art interpretation or the analytical skills needed for musical composition directly boost cognitive abilities. This interdisciplinary approach encourages students to connect disparate fields, promoting a holistic worldview essential for future workforce success. It underscores the value of these subjects in enhancing problem-solving and creative capacities.

Furthermore, the emotional and social benefits derived from art and music are profound, offering unique opportunities for personal expression and emotional intelligence development. These mediums facilitate the cultivation of empathy and the enhancement of social skills like teamwork and communication, preparing students for the collaborative nature of modern workplaces. Additionally, engaging with diverse cultures through these subjects promotes inclusivity and global understanding, critical in our globalized society. This not only prepares students for the professional world but also enriches their personal growth and social awareness.

In conclusion, incorporating art and music into education is essential, enhancing creativity, emotional intelligence, and social skills vital for modern success. These subjects enrich academic learning and are crucial for developing innovative, well-rounded individuals. Therefore, it's crucial for educational systems to prioritize these disciplines, ensuring their significant role in fostering comprehensive learning.

Download IELTS eBooks ,  get everything you need to achieve a high band score

Sample Essay 3

In these days, art and music are deemed to be the compulsory parts of all the communities around the world. In my opinion, it is an impressive path to learn about magnificent lifestyles, such as creative thinking and new life concepts that could give an idea in the innovative digital world. However, they should not spend spare time studying because it could be a detrimental for juveniles to learn useful skills of living.

In the 5th generation network, virtual reality and three-dimensional space have integrated the art and music to show story scenes as if people are living in the real world. Innovative ideas are natural resources to design many artworks and graceful melodies are precious gifts to create new belief in modern life. The movie - avatar is a typical example that depicts people’s effort to raise awareness about the nature through the art and music, this is because the technology combines both factors which seems to repel one another, but it results in a greater creativity. Thus, the art and music, being integrated with the advanced technology, are fully playing their soul nourishing roles even much better than they did it before.

Although the creative activity and theme bring various merits of things to strengthen their spiritual and enhance critical thinking skills, these are not essential subjects for children to study at educational institutions. This is because juveniles should develop useful skills like leadership, problem-solving and trading skills that can heighten better working ability. For example, taking an MBA program is an absolute method to acquire knowledge, the reason for this is that art and music can increase inner strength, but it is unable to improve their technique. This is why young people should take part in all kinds of internships to gain work experience or to satisfy job requirements.

In conclusion, both brilliant art and splendid music are unique ways to enhance physical and mental development. However, children should be exposed to a balanced discipline that ensures a coexistence of art and music, along with a job-oriented skill development approach.

Get your personalised   IELTS Essay Feedback  from a former examiner

  • Task 2 Sample Essays

Related Posts

More and More Wild Animals Are on the Verge of Extinction and Others Are on the Endangered List - IELTS Essay

It is Better to Live in a City, or Life is Better in the Countryside? (IELTS Essay)

Internet has Brought People Closer, or Communities have Become More Isolated Due to the Internet?

Comentários

IELTS Practice.Org

IELTS Practice Tests and Preparation Tips

  • Band 9 IELTS Essays

Here at ielts-practice.org we have a huge collection of band 9 IELTS essay samples. Click on the links below to read our band 9 essay samples. IELTS essay topics tend to repeat. It is, therefore, imperative that you practice writing essays on topics asked in recent IELTS exams. We are adding more essays to this page, so stay tuned.

Recent IELTS writing topics (January, February and March 2024)

  • Some people feel that the private lives of celebrities should not be openly shared by the media
  • Innovation is often driven by the pursuit of profit and economic growth
  • The world has many towns and cities constructed in previous centuries
  • Most modern families have both parents working and as a result children spend less time with their parents
  • Eco tourism often involves visiting remote and fragile ecosystems
  • Some argue that music mainly serves as a way for individuals to reduce their stress and anxiety
  • Some people believe that professional athletes serve as positive role models for people
  • In many societies there is a growing emphasis on individualism
  • In the future, it may be necessary for us to live on other planets
  • In many countries, the number of plants and animals is declining
  • Some people think that certain old buildings are more worth preserving than other ones
  • Nowadays, there is more and more competition for getting into university
  • In today’s digital era anyone with a smartphone can capture and share photographs
  • Everybody should donate a fixed amount of their income to support charity
  • An increase in production of consumer goods results in damaging the environment
  • We have witnessed that parents spend ample amount of money on children’s parties
  • Archeology is partly the discovery of treasures of the past
  • The rise of social media platforms has made it easier for people to vent their frustrations publicly
  • The demand for coaching services has grown significantly in the digital era
  • Some people think that manufacturers and shopping malls should sell fewer packaged goods
  • Financial education should be included as a mandatory subject in schools

Popular essays

  • In recent years, there has been a rise in the use of performance enhancing drugs in sports
  • Advantages and disadvantages of the internet
  • In today’s digital era anyone with a smart phone can capture and share photographs
  • People should be at least 21 years old before they are allowed to drive a car
  • In many countries, formal exams are used to assess students ability and to judge the success of their education
  • Some people say the main environmental problem of our time is the loss of particular species of plants and animals
  • Some people think that the best way to become successful in life is to get a university education whereas others say this is no longer true
  • Success is often measured by wealth and material belongings
  • Completing university education is thought to be the best way to get a good job
  • Some people believe that the typical teaching situation of a teacher and students in the class will not exist by 2050
  • Some people say that ebooks and modern technology will totally replace traditional newspapers and magazines Environmental damage is the problem of most countries
  • Academic Writing Task 1
  • Agree Or Disagree
  • Band 7 essay samples
  • Band 8 Essay Samples
  • Band 8 letter samples
  • Discuss Both Views
  • Grammar exercises
  • IELTS Writing
  • Learn English
  • OET Letters
  • Sample Essays
  • Sample Letters
  • Writing Tips

Enter your email address:

Delivered by FeedBurner

IELTS Practice

IELTS Podcast

IELTS Band 9 sample essay

Band 9 Sample answers are useful as study guides for IELTS preparation for the IELTS Writing Task 2 essay – especially for a band 9 IELTS essay. Having access to previously completed work that you can have confidence in will show you what you are missing!

Take a look at these sample task 2 essay questions to help you prepare for your exam.

Use the following IELTS sample essay and its explanations to see how close you are to a band 9 in your IELTS writing essay!

Evaluation Criteria

Get your IELTS essay evaluated online (free)

Examples of Band 9 Essays

IELTS Writing Task 2 Essay Sample Question and Answer(1)

Why is this IELTS Essay a Band 9?

5 Tips for a Band 9 IELTS Essay

Sample IELTS Writing Task 2 Question and Answer(2)

Sample IELTS Writing Task 2 Question and Answer (3)

Useful Definitions of Advanced Vocabulary Used

Video: Band 9 EX-IELTS Examiner Essay Review

Sample 19 IELTS Essays and 240 Task 2 Essay Questions Ebook

Video: Useful IELTS Essay Writing Tip

Audio Resources

Additional IELTS Resources

IELTS essay task 2: evaluation criteria

IELTS writing tests are evaluated across 4 areas when your band score is calculated:

  • Task achievement  – To what extent does the examinee address all parts of the task with a fully developed position, inclusive of fully extended and well supported ideas?
  • Coherence and cohesion  – Does the candidate logically organise the information and ideas? Is the entire essay cohesive with a logical progression of ideas?
  • Lexical resource  – To what extent does the examinee use a wide range of vocabulary with accuracy? Do they demonstrate sophistication regarding the use of lexical items?
  • Grammatical range and accuracy  – Does the examinee use a range of grammatical structures accurately? Examples of these can be the use of complex sentences with sophisticated clauses instead of simple sentences with a repetitive structure:

Example : Students cannot use phones. They affect development > students are not allowed to use mobile phones in class due to possible distractions.

The British Council (the administrator of the IELTS) outlines 9 different bands of performance for each of the above dimensions here. Your scores in each of these dimensions are averaged to determine your overall band for your essay.

Let's take a look at an example essay that scored as band 9 and then we'll dig into each of these four areas to see why it received that score. It's very important to understand what the IELTS examiner is looking for.

These four criteria are used in our new online essay checker that gives you an estimated band score (free).

IELTS essay sample question (1)

Doing an enjoyable activity with a child can develop better skills and more creativity than reading. To what extent do you agree? Use reasons and specific examples to explain your answer.

IELTS sample essay answer (1)

Parents throughout the world place spend time reading with their offspring to prepare them for school where their literacy skills are further developed; however, recent research suggests that focusing on reading at an early age can be detrimental, and participating in fun activities would be far more beneficial. I am a strong advocate of this approach, and the benefits of it will be covered in this essay.

A fundamental reason for this is that there is no biological age for reading, and pushing infants to acquire this skill before they are ready could have repercussions. For example, in the UK, many boys are reluctant readers, possibly because of being forced to read, and this turned them off reading. By focusing on other activities and developing other skills such as creativity and imagination, when they are ready to read, they usually acquire this skill rapidly.

In addition, the importance of encouraging creativity and developing a child's imagination must be acknowledged. Through play, youngsters develop social and cognitive skills, for example, they are more likely to learn vocabulary through context rather than learning it from a book.

Furthermore, play allows youngsters to mature emotionally, and gain self-confidence. There is no scientific research which suggests reading at a young age is essential for a child's development, moreover, evidence suggests the reverse is true. In Finland, early years' education focuses on playing.

Reading is only encouraged if a child shows an interest in developing this skill. This self-directed approach certainly does not result in Finnish school leavers falling behind their foreign counterparts. In fact, Finland was ranked the sixth-best in the world in terms of reading.

Despite being a supporter of this non-reading approach, I strongly recommend incorporating bedtime stories into a child's daily routine. However, reading as a regular daytime activity should be swapped for something which allows the child to develop other skills.

Why is this essay a band 9?

Task achievement.

According to the IELTS Writing Band Descriptors, an essay is Band 9 for Task Achievement if it:

  • Fully addresses  all parts of the task
  • Presents a  fully developed  position in answer to the question with relevant, fully extended and well supported ideas.

In order to score well on Task Achievement, the most important thing is  to make sure you respond to what is being asked of you . Is the prompt asking for an opinion, a discussion of a problem, a solution to a problem, or some combination of these? If you provide an opinion and not a solution when you're being asked for a solution, you're not going to score well in this area. Read the question carefully!

The prompt for this essay asks:  “To what extent do you agree [with the previous statement]? Use reasons and specific examples to explain your answer.”  It wants an opinion – with support!

This essay addresses all parts of this task. The opinion is included in the introduction to make the writer's position clear, and then the following paragraphs support the writer's position with examples and justifications. Overall, the response is full and relevant and each of the points is detailed and connected to the thesis.

Coherence and cohesion

Think of this as “How well does the essay flow? Is it easy to follow and does it all tie together?” The exact characteristics for a Band 9 C&C score are that an essay:

  • Uses cohesion in such a way that it attracts no attention
  • Skilfully manages paragraphing

Note the specific wording “it attracts no attention.” The goal here is for things to sound natural and not forced. How do you connect your ideas (ensure cohesion) without it sounding forced? I think there are 2 possible ways:

  • Explain your ideas in a logical order so that you don't need many linking words . This is probably what you do when writing in your own language.
  • Use easy linking words like and, but, also, firstly, secondly, finally, for example. These are so common that they attract almost no attention.

This IELTS Sample essay does a good job of this – you'll notice that each paragraph naturally (logically) follows the one prior, providing additional support for the original opinion, and some simple linking words –  in addition, furthermore  (both paragraph 2) and  moreover  (paragraph 3) – are used throughout. These are all good discourse markers that show what is coming next adds to the argument and are slightly more sophisticated than firstly, secondly, and thirdly but don't come across as being forced.

The other aspect to scoring high in C&C is ensuring an essay is well-structured. What do I mean by that? A well-structured essay has a good introduction, body paragraphs that are easy to follow and connect with one another, and a good conclusion. Each body paragraph should also have its own topic sentence and support and then smoothly transition to the next paragraph.

Our sample IELTS essay has a “simple but good” introduction in which it shows that the examinee has knowledge of the topic and clearly states the writer's position to set up the rest of the essay. The paragraphs all have topic sentences, which are then supported by examples, and are easy to follow. The main body and conclusion relate back to the thesis in the introduction.

A note on conclusions…  there are two schools of thought when it comes to how to conclude an IELTS essay. One is to conclude with one simple sentence so that you spend more time perfecting your main body paragraphs. The other is to wrap up with two sentences, once which includes a small prediction (ie, how you think things might turn out) as a way to show the examiner that you know how to correctly use another tense (which will help boost your GR&A score – more on that in a minute). Either is fine, just don't forget your conclusion!

Taking time to plan out and organise your response  before  you start writing is an extremely important step in scoring well in Coherence and Cohesion for your IELTS essay – make sure you do so to ensure your essay is well structured and reads cohesively when you're done!

Lexical resource

Scoring well in the  Lexical Resource  dimension is all about (correctly) showing off your vocabulary. The description for a Band 9 here is:

  • Uses a wide range of vocabulary with very natural and sophisticated control of lexical features, rare minor errors occur only as ‘slips'

Collocations,  topic-specific  vocabulary  and  phrasal  verbs  are the name of the game here. To score well, an examinee needs to show that they have a wide-ranging vocabulary and they know how to use it.

Our sample essay does a solid job of showing off a  range of vocabulary  – you'll notice that while the essay frequently refers to children, the writer employs different vocabulary ( infants, youngsters, offspring, counterparts ) to do so.

Note : it is highly likely that you will need to refer to people/children in your IELTS Writing task 2 , so make sure that you have lots of different words to use to refer to them.

IELTS examiners do not like to see the words  “people,” “children”  over and over again! The same goes for the word “ important ” – make sure you have plenty of alternative phrases ( essential  and  vital  are both used in our sample essay).

Other examples of a  wide-ranging vocabulary  in our essay include using  rapidly  in place of  quickly ,  mature  instead of develop,  repercussions  to indicate a negative result, and  acquire  in place of learn.

Our sample essay also does a good job of using  collocations  – some examples include  “fundamental reason,” “reluctant readers” “social and cognitive skills,” “learn vocabulary through context,”  and  “strongly recommend.”

The correct use of  phrasal  verbs  also demonstrates one's grasp of English – because of the semantics involved, they are sometimes one of the most difficult things for English language learners to master. Our essay writer correctly uses a few of these including “ turned them off”  and  “falling behind .”

One note here: students preparing for the IELTS  often ask if they should use  idioms  (like “you're barking up the wrong tree”) in their essays to further demonstrate their grasp of the language. In my opinion, no, you shouldn't. Idioms are informal by nature and not appropriate for a written essay of this type. Stick with demonstrating your range of vocabulary and your ability to use phrasal  verbs  correctly!

Grammatical range and accuracy

The final scoring dimension is related to grammar and grammatical structures – do you know them and can you correctly use them?

The Band 9 description for grammatical range and accuracy :

  • Uses a wide range of structures with full flexibility and accuracy; rare minor errors occur only as slips

Note that there is nothing in the scoring criteria about including specific tenses or sentence constructions. Your main objective should be to reduce the number of mistakes that you make. An essay that contains no mistakes is likely to get a 9 for grammar, regardless of the types of sentence that it may or may not contain.

If there is a nice mix of long and short sentences in your IELTS essays, you'll meet the grammar requirements. Remember, as soon as you write a “long” sentence you are naturally going to use connectives (linking words), which will make the sentence ‘compound' or ‘complex'. So, don't think too much about the grammar – just aim to reduce the number of mistakes that you make, and try to include a few longer sentences.

Some examples from the sample essay that illustrate the writer's grammatical range and help it easily score as a band 9 include:

  • appropriate uses of modal verbs in the passive voice:  “are further developed,” “will be covered,” “must be acknowledged,” “should be swapped.”
  • “ to focus on ” is correctly followed by an -ing form
  • However  is used correctly with a semicolon before it and a comma after
  • “ because of ,” “rather than,” and are correctly followed by -ing verbs

5 Tips for an IELTS writing task 2 band 9 essay

1. answer what is being asked.

Make sure you read the prompt carefully and answer the essay questions you’re being asked. I can’t emphasise this enough. In order to score well on Task Achievement, you need to appropriately and fully address the task.

2. Plan your work, work your plan.

Plan out your essay before you start writing. What are your main points? What order are you going to make them in? How do they link together? Having a well organised essay is key scoring high marks for Coherence and Cohesion. Many IELTS test-takers will spend up to 10 minutes planning out their essay before they start writing. A few points to keep in mind:

  • Your essay should have 4-5 paragraphs in total and at least 250 words
  • Plan your supporting points so that they don’t go off-topic

3. Write, review, re-write

Write your essay, review it and then “rewrite” it. Don’t focus on getting things perfect upfront – you don’t want to waste 15 minutes trying to come up with the perfect synonym for something and then not have enough time to finish your full essay! Write your essay first (an unwritten essay won’t score well at all!) and then go back through it to see how you can improve it. Some essay questions to ask yourself at this stage:

  • Are there places where you can swap out stronger words for weaker ones in order to improve your Lexical Resource score?
  • Are there places where you can phrase things differently in order to illustrate your Grammatical Range?

4. Where are you falling?

To pass with a Band 9 the reality is you need two sets of skills:

  • Exam skills
  • Language skills

What are exam skills?

Can you plan an effective essay? Quickly? Ideally between 3-5 minutes.

Can you think of enough ideas and examples to put in the essay plan?

Firstly you need to discover which of these skills you need. To do this you get feedback, either from an online IELTS essay checker or for more detailed feedback you can use our IELTS essay correction service .

The main goal is to find out which part of the essay writing process is costing you the most amount of time, points or stress.

Personally, the easiest and fastest way to get these skills is to do an online course specialised in training students with these skills. Here is a good course for that.

5. Better language skills?

A lot of students fail the  IELTS exam or end up with a band in their IELTS writing test that does not meet their requirements. Also, a significant number of students look to Google to search for “IELTS Writing tips” or “Task two tips”. These tips might be helpful but sometimes the real problem might just be in their general language or writing skills.

Writing error-free perfect sentences is probably much more challenging than students think, especially under exam conditions i.e in 40 minutes with immense pressure to pass. These can result in often mixed outcomes with both positive or negative development occurring at one and the same time.

One of the most important ways to improve language skills is to receive feedback. This can be by asking someone to review written work and will expose the positive or negative development mentioned earlier. This is very common and not something that is a negative issue overall.

Have a look at our essay correction service that will review your essays for you and help you improve and pass the IELTS test.

Here is a checklist of what is needed for reaching Band 9, it includes what the examiner wants to see, and what to do to write at a Band 9 level.

Sample IELTS writing task 2 question (2)

Today, the quality of life in large cities is decreasing. Discuss the causes and solutions.

IELTS writing task 2: essay sample answer (2)

The global phenomenon of urbanisation from the beginning of industrialisation to the present day has brought opportunity and prosperity, albeit at a cost in the quality of life. With an increasing city population, the complexity of the challenges also increases for the globe as well as the local community. Therefore, the causes and effects of these on the current generation, as well as possible solutions are outlined below.

The causes for the decrease in the quality of life are paradoxically the prosperity endowed on such metropolitan centres. Their growth is largely due to the increase of opportunities on offer, which in turn increases their attractiveness, essentially they are trapped in a positive self-reinforcing cycle. While such developments have a positive impact on immediate economic objectives, it perpetuates behaviours that can have a negative impact in the long term.

However, this eventually leads to a decrease in the quality of life as the city can experience overcrowding, exorbitant property prices, and increased vulnerability to terrorist attacks. For example, the density of London makes it a more efficient place to attack, when compared to a smaller city such as Bradford.

Therefore, due to continuous growth and prosperity, urban citizens, especially the less well off, often experience a lower standard of living. Even greater than this, are the relevant examples of natural disasters such as recent fires in Australia, which brought about unprecedented weather patterns resulting in the destruction of wild and rare animals. These effects are far from uniform, as they affect different countries in ways unseen by previous generations.

Considering the solutions, greater investment in public transport would ease traffic congestion, as would bike lanes. In theory, this would reduce air pollution, and possibly improve the well-being of the population if they did adopt a more active lifestyle and cycle to work. While these solutions are local, if adopted globally, would affect individuals and many countries alike. A collective effort is needed to use social networks and other media to highlight the negative effect of urbanisation as well as the negative sides of the wider ramifications on the population.

To conclude, while it could be argued that urbanisation advantages outweigh the disadvantages, a wealthy city attracts a large population inflow, which then causes pressure on existing infrastructure and security. Various solutions exist to mitigate such drawbacks, such as social networks being used to raise awareness of such negative impacts on many countries, nevertheless an indefinite solution has yet to be found.

Sample IELTS writing task 2 question (3)

Social media marketing can influence what consumers buy. Do you agree or disagree? To what extent do you agree?

IELTS writing task 2: essay sample answer (3)

Since the introduction of social media applications in the early 2000's the world has become a much smaller place. Social media applications such as Facebook, Instagram and Twitter have become information sources for a majority of the global market.

As such, it could be argued that marketing, which happens to be a source of information accessible on these platforms can influence the consumers who use them. This notion is further aided by the rise in online retail stores that conduct the bulk of their transactions online. This makes it easier for the consumer to purchase from anywhere in the world.

As a consumer on social media, you are constantly bombarded with advertisements of various products that are specifically designed to catch your attention. This means that most of the adverts on your news feeds aren't random and will almost always feature something you have previously searched online or something currently popular or trending. Given the fact that most social media users are young consumers who are influenced by current trends and happenings, these adverts will almost always catch their eye.

The habit of sharing, retweeting and liking also ensure that these adverts get around, quite fast. As such, when an advert does reach your news feed you have already probably seen it on your friend's news feed. The truth is, adverts are a form of information and with the age of the internet, information spreads faster than a wildfire.

Therefore, it only makes sense that in the era and age of technology, globalization and the need to be trendy, social media marketing can influence what consumers buy.

Useful definitions of advanced vocabulary used

IELTS Writing Task 2: Useful definition

Paradoxically

Equivalent sentences

“For example, it is said, the CCTV in London has foiled many potential attacks, and therefore greatly increased the security of its citizens.” Could also be said as:

“Statistics show that CCTV used in London has scuppered many a terrorist plot, massively contributing to the security of its citizens.”

More Equivalent sentences Various solutions exist to mitigate such drawbacks, nevertheless an indefinite solution has yet to be found.

Could also be said as:

A myriad of partial fixes exist for these issues, yet a permanent solution is still out of reach.

There are many methods employed to quell this flow of people, but still a reliable solution has not been discovered.

IELTS writing task 2: vocabulary booster

The highlighted sections in the following paragraph represent key phrases or words relating to this topic. Study this paragraph to expand your vocabulary knowledge on this topic:

The modern urban environment varies considerably depending on both the city that produces it and the individual who perceives it; Each experiencing a unique blend of at least some economic success, varying degrees of localised or wider deprivation and periods of growth and decline. Environmental factors permitting, a city will provide well for its citizens as long as it can properly manage the execution of social policy.

Globalisation presents many challenges for those responsible for the policy as large inflows of people are to be expected in a place of success and therefore opportunity; The ensuing mixing of cultures has far-reaching social consequences that can affect how the city is both presented and perceived.

Considerably Con·sid·er·a·ble (kən-sĭd′ər-ə-bəl) adj. 1. Large in amount, extent, or degree: a writer of considerable influence. 2. Worthy of consideration; significant: The economy was a considerable issue in the campaign.

Perceive Per·ceive (pər-sēv′) tr.v. per·ceived, per·ceiv·ing, per·ceives 1a. To become aware of (something) directly through any of the senses, especially sight or hearing: We could perceive three figures in the fog. 1b. To cause or allow the mind to become aware of (a stimulus): The ear perceives sounds. 2. To achieve understanding of; apprehend: Einstein perceived that energy and matter are equivalent . 3. To regard or consider; deem: an old technology that is still perceived as useful; a politician who is perceived to be untrustworthy.

Deprivation Dep·ri·va·tion (dĕp′rə-vā′shən) n. 1. The/an act or an instance of depriving; Loss . 2. The state of being deprived: social deprivation; a cycle of deprivation and violence.

“The town’s generally miserable appearance led her to perceive it as a place of considerable deprivation.”

IELTS writing task 2: further reading

There are many more writing samples for you to explore.

The BBC has great pages on discursive writing and general writing , also, this video is good for learning how to give examples.

You can even read a sample Harvard essay aimed at preparing students for academic writing.

Remember! Select a text that is appropriate for your level. Choosing the wrong text can result in a loss of confidence and feeling bad never helped anyone to learn anything quickly!

Video: Band 9 ex-IELTS examiner essay review

Click here to Subscribe to the Youtube Channel.

Sign up for the sample 19 IELTS essays and 240 task 2 essay questions ebook

Video: ielts writing task 2-extremely useful sentences.

Optimize Your Writing: Try Our Online IELTS Essay Checker

Writing a good essay for the IELTS is important. You want to get a high score, right? But sometimes, it's hard to know if your essay is good. That's why we made a special tool to help you. It's called the online IELTS essay checker .

How does it work? You put your essay into the tool, it then looks at your essay and tells you what you did right and where you can do better. This helps you learn quickly.

The good news is that our tool can help you save money. Some students pay a lot for classes or books to learn how to write better. But our tool is not expensive. And it gives you fast help.

So, after you read the sample essay on this page, try our online IELTS essay checker . It will show you how to write even better essays.

To sum it up, our online IELTS essay checker is here to help you. It's easy to use and not costly. We want you to do your best in the IELTS without spending too much money. Good luck with your writing!

Additional IELTS writing task 2 resources

  • The University of Manchester Academic Phrasebook provides guidelines and examples of how to introduce essay topics, discuss findings and write conclusions
  • The University of Birmingham Guide to Academic Writing provides tips on paraphrasing, in addition to how to plan, structure and write an essay
  • Use these useful sentences for IELTS Writing Task 2 .
  • This page is good for sample essay topics and answers, also for Task Two.

IELTS writing essay task 2 Sample Band 8 The writing part of your IELTS exam is a great place to score some extra points, especially if you are looking to score within band 8. Here is a task 2 writing sample to help you do just that.

Vocabulary for IELTS Vocabulary is probably the most important part of preparing successfully for IELTS. It is used for both the speaking and writing part of the exam. Click here to view some essential vocabulary.

General essay topics The IELTS exam has a number of general essay topics that span a number of disciplines ad subject matters. To have an idea of what to expect check out our list of general essay topics.

  • Free Essay Band Score Evaluation
  • Sign up to claim your free IELTS materials
  • Jump to Band 7 or it’s Free
  • IELTS Writing Evaluation
  • IELTS Band Score Calculator
  • Book Your Online IELTS Test
  • Sample Topic Answers
  • Useful Sentences
  • Sample Task 2 Questions 2022
  • Introduction to Paraphrasing
  • Model Band 9 Essay
  • Five Band 9 Words
  • Model Band 7 Essay
  • Differences Band 9 vs Band 7 Essay
  • Band 6.5 Essay
  • Academic Collocations
  • Topic Sentences
  • Discuss Both Views
  • Tutorial: To What Extent Essays
  • Paraphrasing Introductions
  • Essay Structures
  • Essay Plans
  • Describe a Pie Chart
  • Using Percentages
  • Map Vocabulary
  • Describe Flow Charts
  • Describe a Bar Chart
  • How to get Band 9
  • AT 1 Sample Questions 2022
  • Describe a Graphic
  • GT Task 1 Questions 2022
  • IELTS Vocabulary
  • Google Play / Podcasts
  • Apple Podcast
  • Android App
  • Task 2 Sample Questions
  • AT 1 Questions

Company addresses: HK Office: BW ENGLISH SERVICES HK Ltd, Unit 2512, 25/F, Langham Place Office Tower, 8 Argyle Street, Mongkok, Hong Kong UK Office: BW ENGLISH SERVICES, 120 High Road, East Finchley, N29ED, London, England, United Kingdom ​+44 20 3951 8271 ($1/min).

IELTS NINJA

Press ESC to close

IELTS Band 9 Essays

IELTS Band 9 Essays: How to Write 9 Band Essays in IELTS

IELTS writing task involves two parts, task 1 and task 2. In task 1, the candidate has to write a summary of a given set of data or diagrams. In task 2, the candidate has to write an essay on a given scenario or problem in above 250 words. This is a crucial part of the writing test and carries 66 per cent marks out of the total writing score. This blog will give you tips on how to write 9 band essays in IELTS.

Getting a band 9 score is a daunting task, but it is not impossible. There are four core areas to be strengthened; task response, coherence and cohesion, vocabulary, and grammar. These are the four aspects on which the writing test is marked, and each one carries equal importance.

What is the IELTS Writing Task 2 Format?

The total duration of the IELTS Writing task is 60 minutes, and it is advisable that task 2 should not take more than 40 minutes. The essay for task 2 should be above 250 words. There is a range of question types which are put for task 2 writing; like opinions, discussions, or pros and cons.  The writing has to be formal, and the question types may differ according to the examiner’s preference.

In the case of General Training, the process is similar. However, the topics for General Training might be a bit more easy and simple than the Academic ones. There is equal weightage and marking for task responses, coherence and cohesion, lexical resources, and grammatical range and accuracy.

Also Read: IELTS Writing Task Evaluation With Sample Answer: A Guide to Better Band Score

IELTS Sample Essays: How to Write 9 Band Intro Essay in IELTS?

Following ahead will be IELTS sample essays for writing task 2. These will also guide you on how to write 9 band intro essays in IELTS. The various types of questions that are asked are:

  • Opinion based essays
  • Discussion-based essays
  • Solution essay questions
  • Advantages/ disadvantages based essays
  • Direct questions

The following can be general topics for IELTS sample essays:

  • Health and wellness
  • Art, language, culture
  • Communication and media
  • Business and economy
  • Family and society
  • Travel and tourism
  • Transport and work
  • Science and technology

IELTS Band 9 Essay Sample 1

What do you think has been the negative impact of the various social media networking sites and apps? Describe both for an individual as well as the society. Social networking sites, such as Facebook, are thought by some to have had a pernicious effect on individual people as well as society and local communities. However, while I believe that such sites are mainly beneficial to the individual, I agree that they have had a damaging effect on the local communities.

Discussing the aspects

With regards to individuals, the impact that online social media has had on each individual person has clear advantages. Firstly, people from different countries are brought together through such sites as Facebook whereas, before the development of technology and social networking sites, people rarely had the chance to meet or communicate with anyone out of their immediate circle or community. Secondly, Facebook also has social groups, which offer individuals a chance to meet and participate in discussions with people who share common interests. On the other hand, the effect that Facebook and other social networking sites have had on societies and local communities can only be seen as negative. Rather than individual people taking part in their local community, they are instead choosing to take more interest in people online. Consequently, people within local communities are no longer forming close or supportive relationships. Furthermore, society as a whole is becoming increasingly disjointed and fragmented as people spend more time online with people they have never met face to face and who they are unlikely to meet in the future.

Ending The Essay

To conclude, although social networking sites have brought individuals closer together, they have not had the same effect on society or local communities. Local communities should do more to try and involve local people in local activities in order to promote the future of community life.

Also Read: 7 Golden Rules of English Grammar: Must-Know Things About Grammar for IELTS

IELTS Band 9 Essay Sample 2

There can be two important aspects of getting a job. One can be completing a degree through college, and the other can be developing soft skills through training and experience. Discuss both aspects and give your take on both. It is considered by some that being a university graduate is the key to securing a good job, while there are others who think that it is better to have experience and soft skills. In my opinion, I believe that having a university education is essential for academic jobs, while soft skills and experience are more useful in business.

Discussing The Aspects

On one hand, many think it is easier for most people to find a good job if they are university graduates with a good degree. In other words, having tertiary education puts people one step ahead of others who do not and this can be the deciding factor in getting a good job. The competition to get into universities and the increasing number of graduates shows just how significant this level of education is for people’s future work opportunities. On the other hand, having work experience and soft skills, such as leadership skills and other interpersonal skills, can also throw the balance in favour of the applicant, according to some. For many positions, there is an overwhelming number of applicants and, therefore, it is often thought that relevant experience in that line of work or having acquired useful soft skills that can be valuable to a company, can put one ahead of the game when applying for a position.

Ending the Essay

Finally, in my opinion, whether needing high level education or skills and experience, depends on the position being applied for. Take for example law, medicine, or teaching, it is impossible to be considered for a position without the required educational background. In contrast, in business, it would be more important for a candidate to have soft skills and experience in that line of business so they can step into a position without further training and be of immediate benefit to the company. In conclusion, getting a good job requires a relevant background either in experience or education depending on the type of work and field. People should make sure they attain the necessary skills or degrees before applying for a job in order to be sure of success.

With all topics discussed, it is clearer how to write 9 band essays in IELTS. The IELTS sample essays for writing task 2 give an insight into the intro lines and how to conclude. One should always remember that the key to getting a perfect band score is consistent practice and effort, and it will surely pay off.

Also Read: Scared of IELTS Writing Task 1 Process Chart Type Questions? Here’s How to do It

Content Protection by DMCA.com

Leave a Reply Cancel reply

Banner

Share Article:

You might also like

Writing high scoring ielts essays

Writing High-Scoring IELTS Essays: A Step-by-Step Guide

IELTS writing mistakes

Common IELTS Writing Mistakes and How to Avoid Them

Linking Words for IELTS Writing Task 2

Linking Words for IELTS Writing Task 2: What is Linking Words in IELTS?

Other stories, describe a book you read that you found useful: ielts speaking cue card sample question, best mba universities in australia: things to know about mba in australia for indian students.

en_US

  • IELTS Twenty20 Course
  • IELTS Model Answers
  • Study Abroad NEW
  • Visas & Citizenship NEW

7 IELTS Essay Samples of Band 9 Students

feel free to call us    +61.4.50973975      [email protected]

Is it even possible to write a Band 9 essay in IELTS?

Well, it is certainly possible. In fact, not only have I scored a band 9 in writing myself, I have also helped several of my students score band 9 in writing too. If you want to learn the art and science behind scoring an IELTS writing Band 9 (or if you just want to improve your score by a few bands), read on.

Every day we receive 100's of IELTS essays for correction from our students. Our experienced IELTS tutors go over every single word of the essay and mark them based on the criteria specified in the IELTS Band Score Descriptors.

Since a lot of IELTS students struggle in the writing section, we thought we would list out the IELTS Band 9 essay samples that we have seen from our IELTS Twenty20 Course students so far. An important thing to note is that the students who wrote these essays went through several feedback rounds with other essay topics where they perfected the art of writing a good IELTS Task 2 essay. So don't get intimidated if you think you cannot write such essays. Everyone struggles with it and it takes time to improve.

But, before we look at the IELTS Band 9 essay samples, let's first understand how to write the perfect IELTS essay.

How to write an IELTS Band 9 essay?

In IELTS Writing Task 2, you are given brief details of an opinion, an argument or a problem, and have to produce an extended piece of discursive writing (an essay) in response.

You need to write at least 250 words and should spend about 40 minutes on this task.

Let's look at a step by step process on how to write a great writing task 2 essay every time ... no matter what the topic.

ielts essay on art band 9

What are the different types of IELTS essays?

Understand the IELTS writing task 2 marking criteria

When IELTS examiners mark your essays they refer to the IELTS writing band descriptors . Here's what the band descriptors mean in plain English.

Identify the main topic of the essay

  • If you incorrectly identify the main idea then you  CANNOT  score above Band 4.
  • If you present a main idea that is not sufficiently developed and supported by examples then you  CANNOT  score above Band 6.

Identify all parts of the task

  • If you address only some parts of the task and not others then you  CANNOT  score above Band 5.
  • Even if you identify all parts of the task correctly but fail to cover each of them fully you  WILL NOT   be able to score above Band 6.

Present a position/opinion

  • If you do not express a clear position then you  CANNOT  score above Band 5.
  • If you do not write a conclusion at the end you  CANNOT  score above Band 5.
  • If your conclusion is unclear or repetitive then you  CANNOT  score above Band 6.

In other words, if you want to score Band 7+, you need to consistently, accurately and appropriately demonstrate the use of all 4 points highlighted above.

Did you think that was all ... nope there's more ...

Even if you do all of the above there is still a chance that you may not be able to score above Band 7. In order to really ensure Band 7+ you need to master  the 4 C’s of Essay Writing .

The 4 C's of Essay Writing

Cohesion  - refers to words and phrases that help link ideas together.For example:

  • Because of this ....
  • It is clear that ...
  • It can be seen ... etc.

Conciseness  - Long sentences do not mean more marks. Run-on sentences will often cause you to lose marks in this area. There are three sentence structures you should be using:

  • Simple sentence  - Contains a subject and a verb and expresses a complete thought. For Example - The teacher returned the homework.
  • Complex sentences  -  Has an independent clause (simple sentence) joined by one or more dependent clauses (cannot stand alone as a sentence) For Example - The teacher returned the homework after she noticed the error.
  • Compound sentences  - Two simple sentences joined by a coordinator (ex. for, and, or, yet, so). For Example - The teacher returned the homework so everyone got to go home early.

Coherence  - How easy is your essay to understand? In order to improve your coherence, proper grammar is a must. You are not there while the essay is being marked, so your ideas need to be clear and easy to understand. Using the cohesive phrases mentioned earlier, can improve the coherence of your essay.

Composition  - The structure of your essay (introduction paragraph, 2-3 body paragraphs, and a conclusion paragraph). A good introduction includes a little background on the topic, a thesis statement, and a preview of the 2-3 main points of your essay. Each body paragraph should include a topic sentence illustrating your point, an example of your point and how it ties into your topic sentence, as well as a concluding sentence that ties this point into your thesis.

Conclusions should reiterate your two or three main ideas from your body paragraphs an restate your thesis again using different words than before. To end your conclusion, you should give a prediction or recommendation on the essay topic.

Note: Remember a proper paragraph has at least 3-4 sentences. Each paragraph should revolve around a main idea, and when you start a new idea, you should start a new paragraph.

How to identify the main topic and all parts of the IELTS essay?

This tutorial will teach you the key steps to identify the main topic and all parts of the IELTS Writing Task 2 question.

Follow the 3 steps mentioned in the video and you will never go wrong.

How to brainstorm and organise your ideas for IELTS writing task 2?

Once you have identified the topic and question parts for your writing task, the next step is to brainstorm ideas that should become part of your essay. In order to get a good band score it is not enough to just create a list of ideas - you need to extend and explain each of those ideas in detail. Lets look at our example from before:

This tutorial will teach you how to brainstorm and extend your ideas for IELTS Writing Task 2 question.

Tips for writing an IELTS band 9 essay

Here's a check list for Writing Task 2. Follow this check list and you are guaranteed to score Band 7+ in IELTS Writing.

  • First off, read & understand the topic of the essay for Writing Task 2.
  • Identify all the key parts of the question.
  • Brainstorm and organise your ideas to ensure that each of your ideas is fully explained and well supported with examples.
  • If the question asks for your opinion, make sure you state it clearly and you don’t contradict that view throughout the rest of the essay.
  • Learn the structure of an essay: Introduction , body paragraphs, conclusion
  • Do not copy the question word for word for your introduction or else those words will be deducted from your total word count. Instead always paraphrase the question in your own words.
  • You MUST write a conclusion/overview at the end. Don’t add new information in your conclusion. Instead, rephrase your key points, and give a strong ending sentence that ties everything together.
  • Always write in a formal tone and use it consistently throughout the essay.
  • Do not use bullet points or short notes.
  • Use a wide range of grammatical structures and vocabulary.
  • Remember to follow the 4 C's of essay writing.
  • Practice and learn synonyms so your writing has a range of vocabulary and does not become repetitive.
  • Write at least 250 words. Anything less, you will lose marks.Ideally the essay should be about 250-280 words.
  • Write neatly, as the person who is marking your essay should be able to easily read and understand what you have written.
  • Do Task 2 first, as it is worth twice as much as Task 1, so priority should be placed here.

Practice makes perfect. Write as many practice essays as you can, and have them marked by an English teacher for mistakes.

  • Practice timing yourself at home, and stick to the allotted time for each section. During the real test, bring a watch and manage your time carefully.
  • Check your writing. If you finish with extra time, look over your essay for any spelling, grammar, or other mistakes you might find.

Popular Topics for Writing Task 2

Topics for IELTS writing task 2 are usually related to some issue or problem that is currently affecting society and you need to discuss it. In recent IELTS exams, topics have mostly dealt with:

  • Environment
  • Animal rights
  • The Internet

Frequently asked questions about IELTS Writing Task 2

Q: Will I lose marks if I write too many words (400-500) in my essay? 

A: There is no penalty for writing more than 250 words for writing task 2. However, there are also no extra marks for writing more. In fact, the more you write, the more you may end up making spelling or grammar mistakes. It is much better to write around 280 words within 35 minutes and spend the last 5 minutes reviewing your work for mistakes.

Q: Will I lose marks if my handwriting is very poor? 

A: In IELTS, handwriting does not affect your scores directly. The scoring rubric does not have any points for handwriting. However, it affects your score indirectly. i.e if your handwriting is illegible, the examiner will think that you have misspelled a word and will mark you lower on lexical resources. The examiner will not give you the benefit of doubt if she is not sure about the words you have written.

Moreover, handwriting also affects the overall impression on the examiner. Remember that IELTS examiners are humans and like all humans they form their first impressions looking at your handwriting. The clearer your work, the better first impression you will make on the examiner.

Click here for recent IELTS exam topics and questions from all over the world

Finally, here are the 7 examples of band 9 essays.

IELTS Writing Sample Essay 1 -Fresh water demand causes and measures

IELTS Writing Sample Essay 2 - Forests are the lungs of the earth

IELTS Writing Sample Essay 3 - Job and money

IELTS Writing Sample Essay 4 - Aim of University Education

IELTS Writing Sample Essay 5 - International Tourism

IELTS Writing Sample Essay 6 - Image is a more powerful way of Communicating

IELTS Writing Sample Essay 7 - Work or travel between finishing high school and starting university

For more IELTS tutorials and feedback for speaking and writing tasks, sign up for IELTS Twenty20 Online Course today! Use Promo Code AUSSIE to get 10% discount at checkout. Study 20 minutes a day for 20 days and Ace the IELTS exam.

Note: we have recently added more ielts band 7, 8 and 9 model answers for speaking and writing tasks. check them out., free ielts level check.

Take our 5-minute IELTS level check and receive a detailed report highlighting the areas where you need to improve the most to achieve your target IELTS score.

IELTS CORRECTION PACKS

bonus

HOT BONUS 1:

Hot bonus 2:, hot bonus 3:, hot bonus 4:, hot bonus 5:, like us on facebook.

Tel: +61 450 973 975 Email: [email protected]

Recent Posts

  • IELTS Model Answer: Countries must invest resources on the young population
  • IELTS Model Answer: Living in big cities is bad for people’s health
  • IELTS Model Answer: Technology makes life complex
  • IELTS Model Essay: All people will choose to talk the same global language in the future
  • IELTS Model Essay: Countries should invest resources on the young population
  • IELTS Samples of Band 7, 8 & 9 Students
  • Study Abroad
  • Immigration Advice
  • Code of Conduct for Australian RMAs
  • Registered Migration Agent (Australia) MARN 2016128

©2012-2024  All prices are in USD. IELTS Online Practice is provided by Wisekangaroo Pty Ltd (ABN: 86 159 373 770)

Send us an email with any questions about our courses and we'll get back to you, asap.

Log in with your credentials

Forgot your details.

ielts essay on art band 9

  • Writing Correction
  • Online Prep Platform
  • Online Course
  • Speaking Assessment
  • Ace The IELTS
  • Target Band 7
  • Practice Tests Downloads
  • IELTS Success Formula
  • Essays Band 9 IELTS Writing Task 2 samples – IELTS Band 9 essays
  • Essays Band 8 IELTS Writing – samples of IELTS essays of Band 8
  • Essays Band 7 IELTS Writing – samples of IELTS essays of Band 7
  • Essays Band 6 IELTS Writing – samples of IELTS essays of Band 6
  • Essays Band 5 IELTS Writing – samples of IELTS essays of Band 5
  • Reports Band 9 IELTS Writing – samples of IELTS reports of Band 9 (Academic Writing Task 1)
  • Reports Band 8 IELTS Writing – samples of IELTS reports of Band 8
  • Reports Band 7 IELTS Writing – samples of IELTS reports of Band 7
  • Letters Band 9 IELTS Writing Task 1 – samples of IELTS letters of Band 9
  • Letters Band 8 IELTS Writing – samples of IELTS letters of Band 8
  • Letters Band 7 IELTS Writing – samples of IELTS letters of Band 7
  • Speaking Samples
  • Tests Samples
  • 2023, 2024 IELTS questions
  • 2022 IELTS questions
  • 2021 IELTS questions
  • 2020 IELTS questions
  • High Scorer’s Advice IELTS high achievers share their secrets
  • IELTS Results Competition
  • IELTS-Blog App

IELTS Essay Samples of Band 9

  • 27 Comments
  • IELTS Essays - Band 9 , IELTS Writing Samples

IELTS Model Essay Samples Band 9, 2023

Last updated: May 3, 2023

Here you can find IELTS Essay samples of Band 9, written by a native English speaker and a former IELTS examiner. Robert Nicholson is the co-author of ‘High Scorer’s Choice’ IELTS Practice Tests book series, created in collaboration with Simone Braverman, the founder of this website. New essays are being added weekly.

Click on one of the topics below to jump to essays on that topic.

Crime and Punishment Education Environment Family and Children Global Issues Government and Laws Health Housing and Town Planning Language Media and Advertising Science Society and Social Matters Sport and Exercise Tourism Work

ielts essay on art band 9

IELTS Model Essays on the Topic of Crime and Punishment

Former prisoners commit crimes after release (reasons and solutions) – Sample essay 1

IELTS Model Essays on the Topic of Education

Schools should teach their students how to survive financially in the world today (agree/disagree) – Sample essay 1 Some people believe that teaching music in schools is vital, while others think it is unnecessary (opinion) – Sample essay 2 Teachers should be required to conform to a dress code (agree/disagree) – Sample essay 3 Many people nowadays travel abroad for their university education (discuss) – Sample essay 4 Some schools insist that students have laptops in class (advantages/disadvantages) – Sample essay 5 Should governments or teachers be responsible for what is to be taught in schools (opinion)? – Sample essay 6 Do schools still need to teach handwriting and mental mathematics skills (opinion)? – Sample essay 7 Should boys and girls be educated separately (opinion)? – Sample essay 8 Should school children be given homework (opinion)? – Sample essay 9 Schoolchildren today take part in short work experience sessions instead of school (advantages/disadvantages) – Sample essay 10 Artificial Intelligence will take over the role of teachers (agree/disagree) – Sample essay 11

IELTS Model Essays on the Topic of Environment

Wildlife population around the world has decreased by around 50 per cent, what can we do to protect wildlife? – Sample essay 1 Increases in fuel prices are the only way to reduce world consumption of fuel (agree/disagree) – Sample essay 2 Ending the world’s reliance on fossil fuels will be a positive development (agree/disagree) – Sample essay 3

IELTS Model Essays on the Topic of Family and Children

Some parents think that children must do house chores (opinion) – Sample essay 1 Is using physical force to discipline children acceptable (opinion)? – Sample essay 2 Women, not men, should stay at home to care for children (agree/disagree) – Sample essay 3

IELTS Model Essays on the Topic of Global Issues

Some people believe that the world’s increase in population is unsustainable, while others think it is necessary and beneficial (opinion) – Sample essay 1 The world today is a safer place and governments should stop spending large amounts of money on their armed forces (agree/disagree) – Sample essay 2 With the scale of globalisation today, it would be best to have a single world currency (agree/disagree) – Sample essay 3 There is a moral necessity today for the richer countries of the world to help the poorer countries develop (agree/disagree) Sample essay 4

IELTS Model Essays on the Topic of Government and Laws

Should the government put a tax on fast food to reduce obesity (opinion)? – Sample essay 1 Some people believe that the problem of illegal drugs can be solved by legalising all drugs (agree/disagree) – Sample essay 2 Unemployment payments encourage people not to seek work (opinion) – Sample essay 3 Individuals should be responsible for funding their own retirement (agree/disagree) – Sample essay 4 Households should have a government-imposed limit on the amount of rubbish they produce (agree/disagree) – Sample essay 5

IELTS Model Essays on the Topic of Health

Some say that people should diet and exercise to lose weight, while others think they should eat better and change their lifestyle (opinion) – Sample essay 1

IELTS Model Essays on the Topic of Housing and Town Planning

Today’s governments struggle to create enough housing for increasing populations while protecting the environment (opinion) – Sample essay 2 In some countries private cars are now banned from city centres (advantages/disadvantages) – Sample essay 3 Is banning cars from city centres a positive or negative development? – Sample essay 4 The advantages and disadvantages of high-rise apartment living – Sample essay 5 Some people like to own their home while others prefer to rent it (discuss) – Sample essay 6

IELTS Model Essays on the Topic of Language

Is learning a foreign language essential or a waste of time (opinion)? – Sample essay 1

IELTS Model Essays on the Topic of Media and Advertising

The number of advertisements for charities is increasing, what is causing this? – Sample essay 1

IELTS Model Essays on the Topic of Science

Breakthroughs in medical science are the most significant advances over the last two centuries (opinion) – Sample essay 1

IELTS Model Essays on the Topic of Society and Social Matters

Some people believe that everyone has a right to access to the Internet and governments should provide it free (agree/disagree) – Sample essay 1 Should copyright materials such as music, films and books be freely available on the Internet (opinion)? – Sample essay 2 Can the society cope with the larger number of elderly people and how? – Sample essay 3 Athletes and entertainers’ enormous salaries reflect our dependence on entertainment (agree/disagree) – Sample essay 4 Is it acceptable that enormous sums are paid for pieces of art when many people around the world live in poverty? – Sample essay 5 Everybody should pay a small amount from their income to help people in poverty (agree/disagree) – Sample essay 6 Only people over 18 years old should be allowed to use social media (agree/disagree) – Sample essay 7 Libraries are not a necessity anymore because of the digital resources available today (agree/disagree) – Sample essay 8 Is fashion a significant part of society, or a waste of time and money (opinion) – Sample essay 9 Traffic on roads has become a problem in nearly every country in the world (solutions) – Sample essay 10

IELTS Model Essays on the Topic of Sport and Exercise

Some people think that sport in schools is a waste of time and resources, while others believe it is a vital part of education (opinion) – Sample essay 1

IELTS Model Essays on the Topic of Tourism

Should governments impose extra taxes to restrict tourism in order to reduce pollution? – Sample essay 1 In some cities the numbers of tourists seem overwhelming, why is this happening? – Sample essay 2

IELTS Model Essays on the Topic of Work

What is the best motivation for workers – salary, job satisfaction or helping others? – Sample essay 1 Having a salaried job is better than being self-employed (agree/disagree) – Sample essay 2 Is studying at university better than getting a job straight after school (opinion)? – Sample essay 3 People today find their lives more and more dominated by their jobs (agree/disagree) – Sample essay 4

Related posts:

  • IELTS Essay Samples of Band 8 Here you can find IELTS Essay samples of Band 8,...
  • IELTS Report Samples of Band 9 Here you can find IELTS Report samples of Band 9,...
  • IELTS Letter Samples of Band 9 Here you can find IELTS Letter samples of Band 9,...

27 thoughts on “IELTS Essay Samples of Band 9”

Dear my online teacher. Do you have a collected sample essays band 8 or 9 in one pdf. I am facing difficulty in writing. I hope you send me within a short time

I need an example of following writing task. If you have, Can you please share it with me. “Some people feel that developments in science are happening so fast that it is difficult for peopleto appreciate the effects of such advances. Others feel we should trust scientistsmore and stop worrying. Discuss both views and give your own opinion”

Hi, how can I write the introduction for this essay?

More people decided to have children in their later age than in the past. Why? Do the advantages of this outweigh the disadvantages?

Please I need your response.

Hi Matthew, in the introduction for this essay you would first describe the problem while paraphrasing the topic (don’t copy it word for word) and you can also say that there are advantages and disadvantages to this decision. You can also include your opinion, or alternatively you can write your opinion in the conclusion paragraph.

Dear my online teacher. Do you have a collected sample essays band 8 or 9 in one pdf. I am facing difficulty in writing. I hope you send me within a short

Worth reading , please update more if you have .

I had been following your update in email , and I am so lucky that I have this site to teach me more strategies in writing an essay.

Great to hear our emails have been useful Barbie! Hope you can use the info to achieve your target score in IELTS.

I have more need of your writing essays because your writing tasks are very convenient or 9 band Plz provide me some eassy regarding all essays …….

Thank you for your feedback Pardeep! It’s great to hear you are finding our model essays useful. We will keep posting them on the blog, you can count on us to help with your IELTS preparation!

Hello can you contact with me if you do not mind? I have some questions and i need your help if you can

Hi Torhijon, how can I help?

Hi I am getting difficult to create ideas in writing all type of essay. How shall I prepare for it.

Hi Samir, are there at least some topics that you don’t have trouble coming up with ideas for? I am trying to understand whether your problem is knowledge-related or not. If it’s knowledge-related, reading well-written essays on a variety of topics can help. Another problem is when people just can’t generate ideas quickly enough, and that is different. Which one do you think it is for you?

That’s great

I’m glad you found the model essays useful! Thanks for your feedback.

If I disagree this idea,in which structure I write Body paragraphs

How can we score 9 band in our ielts exam

If we write little more than 300 words in writing task 2, will we be disqualified? Or get a negative marking?

Hi Sanye, you can write over 300 words in the writing task 2 and you won’t be penalised for that.

Definitely i have hot a lot of questions about writing please help me

Will be happy to, what questions do you have?

Hi! I really appreciate your efforts. Keep sharing such informative stuff. Really amazing. Thank you.

Will do! Thank you for your feedback, Rohan!

I appreciate your sharing. It was a good article. Quite impressive and surely be of great assistance to the public.

Thank you for sharing this valuable information. I really enjoyed reading your post and gained some new insights on many topics. Looking forward to more informative content from you.

Leave a Reply

Your email address will not be published. Required fields are marked *

Save my name, email, and website in this browser for the next time I comment.

TED IELTS

  • A Beginner’s Guide to IELTS
  • Common Grammar Mistakes [for IELTS Writing Candidates]

Writing Correction Service

  • Free IELTS Resources
  • Practice Speaking Test

Select Page

Analysing a Band 9 Sample Answer for IELTS Writing Task 2

Posted by David S. Wills | Jul 16, 2021 | IELTS Tips , Writing | 0

Analysing a Band 9 Sample Answer for IELTS Writing Task 2

Today, I want to analyse a sample band 9 answer to an IELTS writing task 2 question in order to show you what exactly makes an essay successful. Now, there are many things that could make a great essay, but today I just want to show you a few things you might not have thought about before in order to to show you what contributes to an essay getting the highest possible score.

I will start by talking about whether or not sample IELTS essays are actually worthwhile or not, then I’ll group the things I want to talk about into the 4 criteria by which all IELTS essays are judged – that’s Task Achievement, Coherence and Cohesion , Lexical Resource , and Grammatical Range and Accuracy. If you have a special interest in one of these areas, you can skip ahead to that, but I think it’s better to read the whole article.

If you prefer to watch this video, which contains many annotated examples, you can watch it instead of reading this whole article.

Are Band 9 Sample Answers Worthwhile?

Before we begin this lesson properly, I feel I should give a sort of disclaimer. Band 9 sample answers are probably not as useful or important as you might think and there are a few things that you must consider before you use them. Firstly, not all essays that are labelled as “band 9 sample answers” are really good enough to get band 9 in a real IELTS test. As I’ve said many times before here and elsewhere, most IELTS materials are made by people who can’t actually speak English very well and do not understand the exam. These people tend to write essays that are crammed full of words they found in the dictionary but which are incorrectly used. These are absolutely not helpful.

But let’s put them aside for the moment. Let’s say you find a sample answer that really is good enough to get band 9. What can you learn from these? Well, if you use them properly, you can learn quite a lot, but many English learners seem to think of these essays as something they need to reproduce. They tend to view them as the correct way of writing an essay, when in fact it is just one possible example of how an essay could be written.

As such, please note that whether you are using my sample answers or anyone else’s, you should not view them as something you need to copy in any way. Use them instead for inspiration. Maybe you can find some words or phrases there that are helpful, or perhaps the structure is different than in your own essay. You might also find that the writer puts forth their ideas in a very clear way, or there may be some interesting ideas you had not thought of before.

All this is to say, of course, that sample band 9 answers CAN be useful, but that you should not be overly reliant upon them. Considering that, let’s look at our question and answer for today.

A Sample Band 9 Answer

Let’s start by look at the question because all great IELTS essays must start from there. Here is the question we will be analysing:

In many countries today, many highly qualified graduates struggle to find employment. What factors may have caused this situation? What can be done about it?

First of all, let’s think what this question means and what our answer should include. It seems pretty straightforward to me. The fundamental issue is that graduates with good qualifications are finding it difficult to get jobs. We need to talk about why that has happened and what could be done to fix the situation. As you can see, it is a cause and solution question.

analysing ielts task 2 question

To this question, I would give a four-paragraph answer. First of all, my introduction would briefly explain the situation, then I would give an essay outline that said what would come next. My first body paragraph would look at the factors that caused it and the second would pose some suggestions for fixing the problem. Finally, there would be a short conclusion.

ielts writing task 2 structure (cause and solution essay)

Sample Band 9 Answer

Here’s my answer to the above question:

As an increasing number of people go to university, it is now common that graduates cannot find employment. This is a worrying situation, but there are some solutions to it. The current problem of graduate unemployment is largely caused by there being too many graduates. In the past fifty years, going to university has evolved from being an extremely rare occurrence for the smartest students who wished to learn specialist knowledge into something that millions of young people do. The result is a surplus of over-qualified candidates chasing the same jobs. In many countries, the economy is contracting and these graduates have to fight over very few available positions. The ones who do not get a job are left unemployed because they feel too qualified to take a lower-level position and work their way up, especially after having spent four years gaining advanced qualifications. Thankfully, there are some possible solutions. The first thing to do is to recognise that higher education has become too common, and so perhaps it is time that alternative routes are sought. Rather than going from high school to university, students can instead be encouraged to find a vocational training program and enter the workforce soon after leaving school. This would allow them to learn some practical skills rather than the theoretical knowledge they would get from university, and then build up real work experience over a long period of time. In addition, universities could set up better facilities for students to get such practical experience, so that when they look for employment after graduation, they will not be at such a disadvantage. In conclusion, many graduates are unable to find employment nowadays, and there are various reasons for this. The solutions are not easy to apply, but there are some possibilities to reduce the severity of this situation.

Task Achievement

What makes this essay successful in terms of Task Achievement? This part of the test may seem subjective and frustrating because we all have different ideas about issues – particularly ones such as employment and education, which are naturally going to vary from culture to culture. Please note that there are no right and wrong answers in an IELTS exam and even a strange answer could be successful if it was presented intelligently.

The important thing to note from the band descriptors is that your essay must “fully address all parts of” the question and that it is “fully developed” with “relevant, fully extended, and well developed ideas.” One of the biggest problems people face here is cramming too many ideas into their essay. It is impossible to develop your essay sufficiently if you make it into a vast list of supporting arguments. Instead, pick one or two ideas and then develop them.

Let’s look at the second paragraph of my essay. I have picked an idea, which is that there are simply too many graduates nowadays. You might agree with me or disagree with me, but that is not important. Maybe there is a better idea… but again it does not matter because the important thing is that my topic sentence is very clear and each of the sentences that follow it directly support it. The first sentence states my idea in simple terms. The second explains the issue from an education standpoint, with the third making the result very clear – there are too many graduates. The final two sentences tie this to employment and show precisely why having so many graduates causes high unemployment rates.

An examiner might read this and think, “Well, I had a better idea…” but they could not fault me for how mine was explained, and that is key to understanding Task Achievement. It is not about the right idea or the wrong idea; it is about explaining and developing your ideas logically.

Coherence and Cohesion

When it comes to Coherence and Cohesion , there is a lot to consider, but fundamentally it is all about presenting your ideas in a way that guides the reader logically from one point to the next. This means you need a good structure that groups your ideas sensibly and also that you need your ideas to be connected from one clause or sentence to the next.

coherence and cohesion for band 9

I showed you my structure earlier and that is where all good essays should begin. My essay has four paragraphs, which is really all you need for a cause and solution essay – or pretty much any other IELTS essay, for that matter. I devoted one paragraph to the causes and one to the solutions, so that fulfils the requirements for organisation. But structure is easy. You could learn it in a few hours. What is harder is having progression and linking your sentences.

In terms of progression, I explained in the previous section about how each of the supporting sentences in body paragraph developed the topic sentence. This is a good start, but it does not guarantee good progression. I like to think of paragraph structure in terms of making a big statement, then making it more and more specific until you get into the fine details, using examples or hypothetical statements to link the minute details back to the bigger picture.

developing a topic sentence

Doing this isn’t easy. First, you need to think logically and plan your essay, and then you need to use clear language to guide the reader. In my third paragraph, I started with a transitional statement that joined the ideas from the end of paragraph two to the main idea of paragraph three. The main idea started with “The first thing to do…” which is incredibly clear and easy to understand. The reader could not possibly be confused by what I’m saying. Again, my topic sentence presents a broad idea and is followed by another sentence that defines it more clearly. This is followed by a sentence that uses referencing effectively to convey a hypothetical scenario to make my suggestion clear to the reader. I say “This would allow them…” Many people forget that the use of words like “this” and “them” is essential for a good Coherence and Cohesion score.  

how to structure and ielts body paragraph

Finally, I would ask you to look at my essay and tell me how many cohesive devices I’ve used. How many sentences begin with those classic IELTS phrases “On the one hand… on the other hand… for example…” and so on? I would argue that “Thankfully,” “In addition,” and “In conclusion” are the only ones I’ve used. That’s because an essay that intuitively guides its reader by using subtle and intelligent cues will not need many cohesive devices . In fact, it clearly says in the band descriptors that over-using them would cause your score for this section to be around 6 or at most 7.

If you want to understand Coherence and Cohesion fully, then check out the video I made a few weeks ago. It goes into a lot of detail and will help you to understand what is probably the most difficult part of the band descriptors.

Lexical Resource

Whilst Coherence and Cohesion is probably the most difficult part of the band descriptors, Lexical Resource is surely the most misunderstood. That’s because IELTS teachers all around the world are busy telling their students to “use advanced vocabulary.” It is, quite frankly, an idiotic approach and these people are responsible for countless disappointing exam results.

What you need to think of when it comes to IELTS writing is using language in an accurate way and using vocabulary that is topic-specific. Sticking a so-called “advanced” word into your essay should not even cross your mind.

I think most of the people reading this article could look at my essay and understand all of the words I have used. That is because there are no words here that I’ve plucked at random from the middle of a dictionary. However, all of my words are used correctly and that is by far the most important thing. There are also some words that are specific to the topics of education and employment, which is also helpful.

If we look at paragraph three again, we can see some examples of this. The phrase “higher education” is used rather than repeating “university” over and over. “Alternative routes” is used to suggest a different way of going into employment. Importantly, this is a natural phrase that a native speaker would know rather than an awkward expression that has been cobbled together from words we would not intuitively use. The phrase “vocational training program” is excellent because this is a really topic-specific phrase. It falls into the category of “uncommon vocabulary” but you will note that it is not some insanely obscure phrase. This is the sort of language that you should aim for: accurate and relevant. Beyond that we have “workforce” and “theoretical knowledge.”

Importantly, my words are collocated correctly. That means they go together in natural ways. When I say “look for employment after graduation,” it might seem easy to you… You might be surprised this is in a band 9 essay… but a lot of IELTS candidates would write “seek for employment” or misuse the noun “job-seeker” by forcing it into a verb form.

I cannot stress enough the importance of accuracy here. If you want to learn more about Lexical Resource (and I think everyone should), then check out my video on it. This is a deep dive into a profoundly misunderstood subject.

Grammatical Range and Accuracy

Whilst the first three marking criteria have mildly confusing names, Grammatical Range and Accuracy is really very obvious. It is about both range and accuracy.

This means that your essay should use grammar in a correct way (that’s accuracy) but also use different structures (that’s range). You don’t have to go completely over the top and use every single verb tense, clause, and sentence type that your English teacher ever taught you, but it’s best not to sound repetitive.

Going back to paragraph two, we saw a breakdown of the sentences that I used to explain the causes of graduate unemployment. What do you notice about these 5 sentences? It is not obvious when they are written properly in an essay, but when formatted like this, we can see that they vary in length. This is something that good writers do without even thinking about it. Whilst it is not the most important part of an essay, it does affect the reader’s appreciation of it and can impact meaning. Note that the shortest sentence here functions to present a simple point after a very long sentence filled with detail. This is almost like a mid-paragraph conclusion that forces the reader to pay attention.

If we look at the first three sentences, we can see a compound sentence (with the dependent clause first), then a complex sentence, and then a simple sentence. There is a compound-complex sentence at the end of paragraph two, meaning that all 4 sentence types have been included. This is not strictly necessary but it does help in terms of range. I have used relative clauses and switched between active and passive voice where necessary. I have used modals intelligently and all my subjects and verbs are in the correct form. Even my punctuation is right.

This might seem like a very difficult thing to achieve and indeed grammar is the hardest part of the writing test in my opinion. However, it is worth noting that the band descriptors explicitly state that you can make “rare minor errors” and still get band 9, so you can take some encouragement from that.

I really hope that this article has been useful for you. As I stated earlier, sample answers are not always particularly helpful and can sometimes even cause further problems for the people who try to use them. On my website, I have many sample answers that I have written and each of them comes with some description that explains why I have written the essay in that way. I think this is really important because otherwise there is not much you can really learn from them.

are sample answers for ielts worthwhile?

The important thing to take away from today’s lesson though is that, whilst getting a high score for IELTS is certainly not easy, there really is no secret to it. For Task Achievement, you just have to provide a fully developed answer. For Coherence and Cohesion, you have to organise and link your ideas logically. For Lexical Resource and Grammatical Range and Accuracy, you have to make sure that your words are used correctly and that you don’t repeat yourself too much. That’s pretty much the core of it, and anyone who’s teaching tricks and tips is just misleading you.

About The Author

David S. Wills

David S. Wills

David S. Wills is the author of Scientologist! William S. Burroughs and the 'Weird Cult' and the founder/editor of Beatdom literary journal. He lives and works in rural Cambodia and loves to travel. He has worked as an IELTS tutor since 2010, has completed both TEFL and CELTA courses, and has a certificate from Cambridge for Teaching Writing. David has worked in many different countries, and for several years designed a writing course for the University of Worcester. In 2018, he wrote the popular IELTS handbook, Grammar for IELTS Writing and he has since written two other books about IELTS. His other IELTS website is called IELTS Teaching.

Related Posts

Dealing with Difficult Vocabulary

Dealing with Difficult Vocabulary

April 24, 2018

[Listening] Summary Completion Practice

[Listening] Summary Completion Practice

December 18, 2017

Sports [IELTS Speaking Topic]

Sports [IELTS Speaking Topic]

May 24, 2022

IELTS Christmas Vocabulary

IELTS Christmas Vocabulary

December 21, 2020

Leave a reply Cancel reply

Your email address will not be published. Required fields are marked *

This site uses Akismet to reduce spam. Learn how your comment data is processed .

Download my IELTS Books

books about ielts writing

Recent Posts

  • Past Simple vs Past Perfect
  • Complex Sentences
  • How to Score Band 9 [Video Lesson]
  • Taxing Fast Food: Model IELTS Essay
  • Airport Vocabulary

ielts writing correction service

Recent Comments

  • Daisey Lachut on IELTS Discussion Essays [Discuss Both Views/Sides]
  • David S. Wills on Describe a Historical Period
  • Siavash on Describe a Historical Period
  • fabliha on IELTS Speaking Partners
  • tufail khan on IELTS Discussion Essays [Discuss Both Views/Sides]
  • Lesson Plans
  • Model Essays
  • TED Video Lessons
  • Weekly Roundup

Band 9 IELTS Preparation

IELTS General Training

A hub for IELTS GT test takers to help them reach their goal.

How to write an IELTS Essay at band 9 level

This comprehensive article guides you through the processes and techniques of writing an essay that would get you a band score of 9. This is an extensive article, so read it carefully and grasp the concept. Use the examples to understand the concepts explained in it.

In IELTS writing task 2, a candidate needs to write an essay of at least 250 words that examine, support, or oppose a statement through the use of discussions, reasons, logic, arguments, explanations and examples . The essay should aim to state an opinion on a given statement, investigate an issue, discuss a given topic or persuade the readers (the examiner in this case) about something.

Now, an effective way to score well in IELTS writing task 2 is to know where one can gain or lose marks. The essay will be evaluated based on the four marking criteria, and each criterion offers 25% of the total score. Hence, let’s have a look at the assessment criteria first.

IELTS Essay has 4 assessment criteria. They are: 1) Task Achievement 2) Coherence and Cohesion 3) Grammatical Range and Accuracy 4) Lexical Resource

1) Task Achievement:

This criterion assesses the extent to which a candidate can meet the requirements of the task. So, the candidate should sufficiently address all the parts of the question. In other words, he or she ought to form a well-organized response to the task with relevant, supported and extended notions. This can be achieved by having a clear understanding of the task. Otherwise, it will lead to an inaccurate answer. The best approach to do this is to interpret the question correctly.

Task Achievement Dos:

  • Answer the specific question being asked, not the general topic. [The essay question often includes a general topic or statement at the beginning of the essay which is not a part of the real essay question. You should find out what the essay topic is asking you to write about.]
  • Make sure the ideas are directly relevant to the question. [The ideas and arguments you present in your essay should always be relevant and never off-topic.]
  • Properly address each part of the question. [If the essay asks you two questions, for example, address them both with proper examples and explanations.]
  • State the opinion in the introduction and employ supporting paragraphs to support this opinion. [If the question asks you to give your opinion, simply state your position at the end of the “Introduction” paragraph.]
  • Develop key ideas with explanations and examples. [Yes, your essay must have examples and explanations to achieve a higher band score.]
  • Reiterate the opinion in conclusion, or write a solution or generally accepted point or statement.

Task Achievement Don’ts:

  • Devote lots of time to just one part of the question.
  • Provide very general examples.
  • State opinion until the last sentence of the “Conclusion” paragraph.
  • Repeat the same points again and again.
  • Write under 250 words.

Now, let’s look at a bad example first:

Bad example:

Some experts believe that it is better for children to begin learning a foreign language at primary school rather than secondary school.

Do you agree or disagree?

“Learning a foreign language is crucial for people. Firstly, it boosts brainpower. Secondly, it improves performance in other academic areas. Finally, a second language improves memory.”

Now let’s illustrate why it is a bad example:

The writer introduces three main ideas in this paragraph. But, none of them is well-developed. None of them has explanations and examples as well. For instance, the candidate does not explain how a foreign language may boost young learners’ brains. What is more, you also probably have noticed that it talks about “people”. Although the question is about young students, the author writes about general people. He or she does not write an answer to the specific question. So, the writer has not fully developed his/her ideas. Also, the ideas are irrelevant. So, he/she may get only a 5-5.5 band score for this paragraph.

Let’s look at a good example now:

“Learning a foreign language helps young students with their intellectual development. It has been shown that it supports a child’s brain development, and in fact, helps with subjects like physics and mathematics to a great extent. For instance, a recent study, conducted by the University of Cambridge, has revealed that children who learn a foreign language are more likely to be good at physics.

Let’s explain why it is a good example:

This is a good example mainly because the idea is relevant and specific. The idea is also well-developed. So, it has one main idea that is “learning a foreign language helps young learners with intellectual development.” Intellectual development means that young learners become smarter. In addition, it explains that studying a foreign language helps the brain to understand physics and mathematics in a better way making them good at science. It hasn’t stopped here. It develops the main idea even more by giving a specific example that children who study a foreign language are very likely to do well in physics.

Another good example of the same topic is given below:

Furthermore, it is evident that pupils who start learning a foreign language during their childhood, gain mastery of the language in their adulthood. It helps them communicate effectively and showcase their learning and speaking skills on the one hand and enhances their career prospects in the future on the other hand. To illustrate, a recent study by the University of California reveals that learners who start a foreign language like English, French, German or Spanish at a young age do better academically in their colleges and also show better mastery of these languages in their adulthood. And this mastery of these languages helps them get many competitive advantages in their future career.

This is a good example because it includes a relevant and specific idea at the beginning of the paragraph. The idea is then well-developed by the writer with an explanation. It also gives an example.

Interpreting the IELTS writing task 2 (Essay) question:

Interpreting the meaning of an essay question is crucial to writing a good essay in response. When analysing the question, the candidate should first shed light on these three properties: “Topic words” , “Qualifying words” , “Instruction words” .

To demonstrate these three attributes, take the following essay question:

Some people think that it is better to educate boys and girls in separate schools. Others, however, believe that boys and girls benefit more from attending mixed schools.

Discuss both these views and give your own opinion.

The candidate can use the topic words to identify the “subject” of the question. In this essay question, ‘boys’ , ‘girls’ , and ‘the school’ are the three controlling topic words. Topics outside the boundary of boys, girls and school thus have no place in this essay.

Qualifying words tell the candidate how the various notions relate to each other. These words often describe the capacity of the opinion and shape some important thoughts as well. In the above sample question, the phrases “better to educate” and “benefit more” are used. Notice how these phrases clarify what is being said about boys and girls. Likewise, these words separate and mix the qualifying-topic word “school” . Notice how these words mould the question and give it enhanced precision. It’s crucially important for the candidates to understand the degree of this precision to ensure the response is properly aligned with the question.

Instruction words are the words that elicit a response from the candidate. Put simply, these words tell the candidate exactly what to write about and eventually channel the candidate’s essay towards a discussion or argument essay structure. In the above example essay question, the instruction words are – “Discuss both these views and give your own opinion” – a phrase that instructs the candidate to expound both the viewpoints and then venture his/her opinion.

2) Coherence and Cohesion:

“Coherence” in a written piece of work means that the reader can readily understand it. Put simply, coherence is a quality that ensures all the parts of an essay are logically arranged, well-connected and all head to the central focus of the essay. Without coherence, an argument may not make sense or may be difficult for the readers to get the message of the text. It’s a critically important quality of IELTS writing task 2/ IELTS Essay.

Similarly, “cohesion” is how a written piece of work relates its ideas to develop a clear relationship and logical progressions among them. In simple words, it relates to the linking of notions within a sentence, the linking of sentences (links between sentences) within a paragraph, and ultimately the linking between paragraphs.

The coherence and cohesion part of the candidate’s marks is a measure of how logically an essay’s ideas are arranged and connected by the candidate, and how smoothly these ideas flow together. The candidate can achieve coherence by using grammatically correct and short sentences that are concise, and to the point. Cohesion can be maintained by employing phrases or linking words, often called “cohesive devices”, to establish relationships between sentences and paragraphs in the essay.

Below is a list of various cohesive devices and an example of their use. Also, notice the corresponding punctuation:

To maintain sequence:

  • Firstly , separate schools are more relaxing for both girls and boys as they do not have social pressures to worry about, such as impressing the opposite gender.
  • Secondly , separate schools provide the teachers with an opportunity to tailor the curriculum in a way that students can easily connect with.
  • Moreover , separate schools have lesser cliques.
  • Finally , separate schools have a more conducive environment for concentrating on studies.

To provide evidence:

  • For instance , many studies reveal that students in separate schools feel more relaxed than their counterparts.
  • For example , teachers can maintain harmony in the classroom easily.
  • A case in point is the top-ranked schools in our country that are mostly single-gender educational institutes.
  • Take the example of separate schools in India, which have fewer cliques.
  • Many high school students in separate schools, such as those from Asian countries, are good at concentrating on their studies.

To show similarity:

  • Similarly , many high school students in Italy experience grade improvements when educated in separate schools.
  • Likewise , many girls in Indonesia can concentrate more on their studies when educated in separate schools.
  • In tandem with this are the experiences of many boys in Pakistan, whose grades improved when educated in separate schools.
  • Many girls in Malaysia also experienced grade improvements when educated in separate schools.
  • Coupled with this is the experience of many boys in Latin America, whose grades improved when educated in separate schools.

To demonstrate contrast:

  • However , studying in a mixed school impacts positively on the personality of both boys and girls.
  • On the other hand , a mixed-gender classroom provides an environment where girls and boys change their behaviour positively.
  • Many girls and boys in India change their behaviour positively when studying in mixed-gender schools; however , this is not the case in Australia.
  • Many girls and boys in Nepal improved their grades when studying in a mixed-gender classroom, but this is not the case in Germany.
  • Conversely , many girls and boys in India promote sexual equality when studying in mixed-school.
  • It is argued by many people that boys and girls are taught effectively when studying in single-gender schools, yet others believe that boys and girls gain considerable advantage from studying in a co-education system.
  • In contrast , many boys and girls respect each other when studying in the co-education system.

To amplify or extend:

  • Moreover , students in mixed schools improve their grades significantly.
  • Furthermore , several independent studies show that students who are taught in mixed schools prevent gender discrimination.
  • To add to this , a recent body of high-quality research has revealed that students studying in mixed schools perform far better in academics than students studying in single-gender schools.
  • In addition to this , students studying in mixed schools can be more candid than their counterparts.

To present a result:

  • Consequently , the link between the education system and students’ performance is obvious.
  • In mixed schools, girls need to spend most of the day with boys and vice versa. As a result , they learn how to live harmoniously.
  • Thus , students studying in mixed schools change their behaviour positively.
  • Therefore , boys and girls respect each other.
  • Because of this , it can lead to distraction to study.
  • It is clear that there is a close link between the education system and students’ performance.

To draw the conclusion:

  • To reiterate , the co-education system acts as a clear precursor of gender equality.
  • To conclude , the co-education system helps promote gender equality.
  • In conclusion , the co-education system advances gender equality.
  • All things considered , the co-education system is committed to achieving gender equality.

Let’s look at an example and go through some of the features that make an essay coherent. Although it is an entirely subjective judgement, most readers would agree that this is a reasonably coherent paragraph:

The co-education system confers both advantages and disadvantages. It goes without saying that in the presence of the opposite gender, one’s behaviour changes. A case in point is Indian boys, they change their behaviour in presence of girls. Thus , the co-education system reaps such benefits where boys or girls change their behaviour positively. However , this advantage comes at a high price: distraction. Students in mixed-gender schools may think of having a partner. Such thoughts can cause distraction as girls or boys may get attracted to the opposite gender, and if anything goes wrong, it can result in hampering learning, thereby distracting academic studies.

  • Topic sentence: the paragraph begins with a clear, declarative topic sentence that expresses the controlling idea and the rest of the paragraph follows the idea. That is to say that everything in the paragraph deals with the advantages and disadvantages of the co-education system.
  • Key terms (marked in blue colour) : the term “co-education system” is repeatedly introduced in this short paragraph. This shows the reader that the paragraph is about the “co-education system”.
  • Clear transitions (marked in orange colour) : each sentence flows into the next very easily, and the reader can easily see how logically the ideas are organized and connected.

Coherence and Cohesion Dos:

  • Have at least four-paragraph structures. [It means your essay should have at least 4 paragraphs including the “introduction” and “conclusion”.]
  • State main ideas and your opinion in the introduction.
  • Introduce a clear topic sentence in the supporting paragraphs.
  • Use cohesive devices accurately and appropriately.

Coherence and Cohesion Don’ts:

  • Introduce background statement in the introduction.
  • Produce lots of ideas in one paragraph.
  • Use cohesive devices at the beginning of every sentence.

Let’s look at the bad example:

Learning a foreign language is crucial for people. Moreover , it boosts brainpower. Furthermore , it improves performance in other academic areas. Additionally , a second language improves memory. Last but not least , a recent study has revealed that children who learn a foreign language are more likely to be good at physics.

Let’s explain why it is a bad example:

If the candidate uses these words in red called cohesive devices again and again and uses them at the start of every sentence, it demonstrates to the examiner that the candidate doesn’t have good skills in English and he/she is not able to link sentences together without cohesive devices. Besides, “last but not least” is not accurate. Because it is used to give an example. The phrase ‘last but not least’ indicates the last point.

So, let’s look at a good example:

Foreign language helps young learners with their intellectual development. It has been shown that this can help a child’s brain development, and in fact, help with subjects like physics and mathematics. For instance , a recent study has revealed that children who learn a foreign language are more likely to be good at physics.

Let’s clarify why it is a good example:

In this paragraph, the writer employs just two simple cohesive devices. The writer actually gets more marks for using them accurately. This is because a good writer uses simple cohesive devices rather than merely using “moreover”, “furthermore” and so on without context. This paragraph also uses the appropriate cohesive device when giving examples.

3) Grammatical Range and Accuracy:

In the IELTS writing task 2 test, one of the most important areas for a candidate to master is “grammatical range and accuracy”. According to IELTS band descriptors, the test taker has to use a wide range of sentence structures as well as advanced verb forms including modal verbs, conditionals, and passive voice to get a higher band score. Besides, in IELTS writing task 2, punctuation has a profound significance. Put simply, in order to get a high score for grammar, the writing has to be complex, but it also has to be correct. So, devote full attention to those commas, semicolons and other grammar rules.

Grammatical Range and Accuracy Dos:

  • Write error-free sentences.
  • Use a variety of sentence structures.
  • Use complex sentences but use them correctly.

Grammatical Range and Accuracy Don’ts:

  • Try to force too many different structures.
  • Try to dazzle the examiner with complex grammar but make mistakes.
  • Write sentences that hinder the meaning being conveyed i.e. stop the examiner from comprehending and guessing the meaning.

Let’s look at an example:

“In the developed world, carbon emission is one of the most debated issues causing global warming and environmental challenges arising from its catastrophic consequences.”

This is a typical sentence from an essay that is trying to be highly complex. The writer has tried to put four simple notions into one sentence and the result is an incoherent and difficult sentence. The writer fails to maintain control of grammar, thus affecting its meaning. If the meaning is affected, the text becomes difficult to understand and that has a negative impact on the band score.

Let’s look at the example again. There are four simple notions that we can write in simple sentences:

  • Carbon emission is a much-debated issue in the developed world.
  • Carbon emission causes global warming.
  • There are environmental challenges associated with global warming.
  • Global warming has catastrophic consequences.

But, if we write all the sentences like this we fail to score higher marks in the IELTS exam. Actually, we need to put them together to form complex sentences. For example:

“One of the most debated issues is carbon emission which causes global warming. There are environmental challenges associated with this problem and its effects have catastrophic consequences.”

Here, four ideas have been put together in two complex sentences. It has been done to make them easy to understand. This way maintaining grammatical range and accuracy become easier.

4) Lexical Resource:

The “lexical resource” is another important area that an IELTS candidate has to pay attention to. That is to say that he or she should use a good variety of vocabulary in order to achieve a high band score in the writing test.

Let’s break this down into small points:

  • Range of vocabulary – The candidate should use a wide range of vocabulary to convey precise meanings. Synonyms can help here. But keep in mind that every word should be used in proper contexts.
  • Correct spelling – Spelling serves a crucial role. So, be sure to use them correctly.
  • Appropriate use of collocations – These are the natural combination of words in a sentence. In order to ace the IELTS writing test, the candidates should learn not only the individual words but also focus on what other words they are used in combination in a sentence.

Lexical Resource Dos:

  • Be careful with grammar and spelling.
  • Use words that are directly relevant to the topic.
  • Be aware of collocations. Use appropriate collocations.

Lexical Resource Don’ts:

  • Repeat the same words over and over.
  • Force complex words into the essay without knowing them accurately.
  • Use wrong synonyms.
  • Learn lists of academic words out of context.
  • Use rare phrases and jargon.
“Research has evidenced that children who have excess screen time are more likely to be myopic .”

This is a common sentence from an essay that has used the medical jargon: myopic. The writer has tried to impress the examiner by inserting the word “ myopic “. This is tricky because jargon includes the words or phrases used in a particular profession, which are difficult for other people to understand. Consequently, the candidate doesn’t get a higher score in the IELTS Writing exam.

It would be far better if the writer uses a simple phrase/word than a very rare phrase or jargon like ‘myopic’. For example:

“Many research has outlined that children who have excess screen time are more likely to have bad eyesight.”

A common phrase ‘bad eyesight’ has been used instead of a less common medical term ‘myopic’. This conveys meaning accurately while making the sentence easy to understand. So, keep it simple and avoid the use of jargon.

Now that you know what criteria are important to reaching a band 9 level IELTS essay, let’s explore some steps to enhance your band score from 5/6 to 8 or 9.

We would now explore a step-by-step guide to understand the process of writing an excellent essay to get a higher band score.

A step-by-step guide to writing a top-notch essay:

Step 1 – analyse the question:.

A candidate has to analyse the question first so as to understand it. In many cases, candidates do not answer the question fully due to their lack of understanding of the question. Consequently, it stops them from achieving a score higher than band 6.

To analyse the question, you have to identify the question type first, then identify topic words, qualifying words and instruction words, one by one. This will help you understand exactly what the question asks you to do. However, the discussion on how to analyse the question has already been explained above.

Step 2 – Plan The Structure:

Candidates who get a higher mark in IELTS writing task 2 always plan their arguments and ideas. A good plan helps them organise their ideas, and then structure their essay before they write it, saving their time and helping them to write a well-structured, coherent essay. The following tasks should be in the planning phase:

A) Creating the structure plan: → Introduction → 1st body paragraph → 2nd body paragraph → Conclusion

B) Generating ideas: Sometimes it proves to be difficult to write a good IELTS essay due to the lack of good ideas. However, there are several ways to overcome this problem. They are: ✓ The common topic familiarization ✓ Mind mapping ✓ Brainstorming ✓ 6 question method ✓ Thinking informal

Anyway, “practice” is the silver bullet for this problem. The more one practices with different topics the more she or he will be able to produce simple and better main ideas in just seconds. In fact, the candidate can be an idea-generating device through practice. So, practice, practice and practice.

Let’s take a look at the example question and create the skeleton plan:

Structure plan: → Introduction: paraphrase the topic and express opinion [i.e. boys and girls reap more benefits from attending mixed schools.] → 1st body paragraph: it is better to educate boys and girls in separate schools. → 2n body paragraph: boys and girls benefit more from attending mixed schools. → Conclusion: re-paraphrase the introduction and opinion. Alternatively, you can suggest something that would be a good solution to the debate or something that would be accepted by most people. [Example: It is expected that schools teach boys and girls together so that they become responsible and sympathetic individuals in the future who do not allow any gender discriminations.]

Step 3 – Write the introduction paragraph:

The introduction paragraph performs as a roadmap for an essay. It brings up the topic, the writer’s position, and the main points that will be used to strengthen and prove this position. Thus, when a reader reads the introduction of an essay, he/she should know exactly what the rest of the essay will look like.

However, many candidates often produce an introduction with a few common mistakes in them. The common mistakes are:

i) Introducing hooks or long general background statements about the topic. In most cases, essays begin with ‘In modern life……’ or ‘Nowadays….’ followed by general information about the topic. It’s a poor start you can probably make. So, it’s worth bearing in mind that you are asked to answer the question not talking generally about the topic.

ii) Not outlining opinion or main ideas. This is a grave mistake. You should combine opinion with main ideas. In fact, it’s the most important part of the essay that will tell the examiner what you’re going to write in the rest of your essay. If you don’t state your opinion clearly, you will lose marks substantially.

iii) Copying the question. If you just copy the question fully, the examiner will delete or discard it, and you will not get marks for this part of the exam. You can copy some words of the question, but don’t copy the whole question.

Bad and good examples:

Some people say that dangerous sports should be banned since they are deadly and life-threatening.

Do you agree or disagree with this viewpoint?

An example of a bad “Introduction”:

Nowadays dangerous sports are undeniably is a very controversial issue and some people say that they should be banned. This essay will discuss it and then come up with an opinion.

As you notice a bad example starts with a very common style of introduction, copies phrases and words from the question and doesn’t state an opinion or outline statement. So, the three essential elements are not included in the introduction.

An example of a good “Introduction”:

It is argued by many that extreme sports should be prohibited because they prove fatal and life-threatening. I agree up to a point, but I also think it is the democratic right of an individual to choose whatever sporting activity he or she wants to take part in.

In this “example introduction”, the writer paraphrases the question statement by using synonyms and outlines the opinion and main points. The is a great “introduction” because it meets the three obligatory criteria.

Skeleton of a Good Introduction:

It is very crucial that your introduction is well-structured. In order to do that, you have to include three essential things: A. Paraphrase B. Opinion (answer to the question) C. Outline of the main ideas

You ought to combine these three elements into just two sentences and your introduction should be around 40-60 words. Your introduction shouldn’t be longer than that. Don’t spend too much time writing the “introduction” of your essay.

A) Paraphrase Question: Paraphrasing means repeating the question statement in order to convey the same meaning, but with different words and styles. We can do this by using synonyms, different forms of the same words, and rearranging the clauses.

A higher salary is much more important than job satisfaction.

Paraphrase:

It is argued by many that it is beneficial to land a high-paying job, even if it does not content you at all.

As you see in the paraphrased sentence, some words and phrases have been changed while retaining the same meaning. Instead of ‘higher salary’ , for example, ‘high-paying job’ has been used. Thus, it demonstrates to the examiner that the writer can use a wide range of vocabulary and rephrase the question statement correctly.

B) Opinion (answer to the question) : This sentence is considered to be the most important part of your essay. It demonstrates to the examiner that you’ve understood the question perfectly and will drive to a clear and coherent essay.

Let’s look at the opinion sentence from the previous example:

Here, we’ve two choices- Choice 1: This essay totally agrees with this statement. Choice 2: This essay completely disagrees with this statement.

Opinion (answer to the question): This essay profoundly disagrees with the notion that higher remuneration is more crucial than career satisfaction.

The above sentence clearly states the position of the writer.

C) Outline of the main ideas: Outlining main ideas is another crucial thing in your IELTS essays. Through this, you tell the examiner what you are going to discuss in the main body paragraphs.

Plan: Why I disagree: Job satisfaction brings a sense of fulfilment. Doing what people like keeps them motivated, thus leading to a successful career. Combining opinion and main points: This essay profoundly disagrees with this statement because job satisfaction brings a sense of fulfilment and leads to a successful career as well.

In the above sentence, we simply use the word ‘because’ to combine them.

Putting it all together: Question:

“It is argued by many that it is far more beneficial to land a high-paying job, even if it does not content us at all. This essay profoundly disagrees with this statement because job satisfaction brings a sense of fulfilment and leads to a successful career as well.”

As you can notice, the above “introduction” follows the skeleton that has been outlined above. The rephrased question statement is given in green . The opinion has been given in purple , and the outline of the main ideas is in blue . Thus, this is a great “introduction” with two very simple sentences.

Now, we will through the introduction writing techniques for each easy type:

1. Opinion (Agree or Disagree) Essay Type 2. Advantages and Disadvantages Essay Type 3. Discussion (Discuss both views) Essay Type 4. Problem/Causes and Solution Essay Type 5. Double Question/ Mixed Essay Type

For each type of essay, the “introduction” would be slightly different. Let’s look at the examples for each of the five types of essay questions:

1. Writing Introduction for Opinion (Agree or Disagree) Essays: Question: Some people believe that children should be taught how to manage money at school.

Paraphrase: It is argued by many that schools ought to teach students financial literacy.

Answer to the Question: a) Plan- agree . b) Why I agree: ✓ Lack of financial literacy leads to disastrous consequences. ✓ Helps reach smart financial decisions. c) Opinion and Outline of main points: This essay entirely agrees with that statement because children can make financial decisions carefully, understand basic money management, and lack of financial literacy leads to disastrous consequences.

The whole introduction:

It is argued by many that schools ought to teach students financial literacy. This essay entirely agrees with that statement because children can make financial decisions carefully, understand basic money management, and the lack of financial literacy leads to disastrous consequences. (41 words)

2. Writing Introduction for Advantage and Disadvantage Essays: Question: It is suggested by many that young children should learn how to grow vegetables and keep animals.

Do you think that the advantages of this outweigh the disadvantages?

Paraphrase: Many people argue that young children ought to acquire basic knowledge of farming such as cultivating vegetables and keeping cattle.

Answer to the Question: a) Plan – Agree that advantages outweigh disadvantages. b) Advantages and Disadvantages Advantages – children can develop good work ethics and soft skills through gaining such farming knowledge. Disadvantages – children get exposed to hazardous pesticides and herbicides and can be distracted from academic studies.

c) Opinion and outline of main points: This essay will argue that despite children getting exposed to hazardous pesticides and herbicides, and being distracted from academic studies, the work ethics and soft skills they develop mean that the benefits outweigh the adverse effects involved in the process.

Many people argue that young children ought to acquire basic knowledge of farming such as cultivating vegetables and keeping cattle. This essay will argue that despite children getting exposed to hazardous pesticides and herbicides, and being distracted from academic studies, the work ethics and soft skills they develop mean that the benefits outweigh the adverse effects involved in the process. (60 words)

The writer here has used ‘despite’ to tell the examiner that many people think so, but he/she believes otherwise.

3. Writing Introduction for Discussion (Discuss both views) Essays: Question: Some people think that zoos are cruel and should be closed down. Others, however, believe that zoos can be useful in protecting wild animals.

Discuss both views and give your opinion.

Discussion essays introduce two sides of an argument. So, the introduction should be written differently than an opinion essay.

Paraphrase: It is considered by some that zoological gardens help preserve endangered species, yet there are others who regard them as inhumane and opine that zoos ought to be abolished.

You can see two phrases: “it is considered by some” and “yet there are others who regard”. So, there are always two opinions.

Answer to the Question: a) Plan: ✓ Zoos are beneficial because of the breeding programmes for vulnerable species. ✓ Zoos should be abolished because of unnatural environments and cramped cases. b) Opinion and Outline of main points: This will argue that although the breeding programmes contribute significantly to protecting vulnerable species, the writer is of the opinion that zoos should be banned due to inhumane animal conditions.

It is considered by some that zoological gardens help preserve endangered species, yet there are others who regard them as inhumane and opine that zoos ought to be abolished. This will argue that although the breeding program contributes significantly to protecting vulnerable species, I am of the opinion that zoos should be banned due to inhumane animal conditions. (58 words) .

As you can notice that by stating ‘although’, the writer recognizes that some people think that zoos are inhumane and should be abolished, but he/she doesn’t think the same way. However, we’ve introduced both views, answered the question and outlined our key points. So, it meets three essential criteria for a good introduction.

You must bear in mind is that you shouldn’t just write “This essay will discuss both views and then come to a reasoned conclusion” . The reason is that you haven’t expressed your opinion and also haven’t outlined your main ideas. Consequently, you will lose marks.

4. Writing Introduction for Problem/Causes and Solution type essay: Question: In some parts of the world, the rate of divorce has increased dramatically over the past few decades.

Explain some possible reasons for this problem and suggest some solutions.

Paraphrase: Divorce, in many countries, has reached epic proportions over the few decades.

Answer to the Question: a) Plan: b) Problem & Solution: Problem – Domestic violence and gender inequality. Solution – mutual understanding and conjugal appreciation.

c) Opinion and Outline of main points: This essay will expound on how domestic violence and gender inequality are the main reason for increasing separation rates in many parts of the world, followed by a discussion on how mutual understanding and conjugal appreciation are the most effective remedy for this issue.

Divorce, in many countries, has reached epic proportions over the few decades. This essay will expound on how domestic violence and gender inequality are the main reason for increasing separation rates in many parts of the world, followed by a discussion on how mutual understanding and conjugal appreciation are the most effective remedy for this issue. (56 words)

5. Writing Introduction for Double Question/Mixed Essay type: Question: Fossil fuels are essential for producing electricity, powering industry and fueling transportation. However, one day we will reach a point when all the world’s fossil fuels have been depleted.

How can we conserve these resources? What are some alternatives to fossil fuels?

Paraphrase: Energy production is heavily dependent on oil, coal and natural gas but there will come a time in human history when these resources will be exhausted.

Answer to the Question: a) Plan: b) Conservation and Alternatives: Conservation – choosing zero-carbon transportation, and reusing and recycling products. Alternative source – renewable energies: solar, wind and tide power.

c) Opinion and Outline of main points: This essay will argue that we can preserve non-renewable energy sources by choosing zero-carbon transportation, and solar, wind and tide energy are viable alternatives to natural resources.

Energy production is heavily dependent on oil, coal and natural gas but there will come a time in human history when these resources will be exhausted. This essay will argue that we can preserve non-renewable energy sources by choosing zero-carbon transportation, and solar, wind and tide energy are viable alternatives to natural resources. (53 words)

Step 4 – Write The Main Body Paragraphs:

Main body paragraphs/ supporting paragraphs contain the main discussion of your essay. In other words, these paragraphs exist to help prove your position by employing real and factual – or seemingly real and factual- information. Therefore, this is where you can gain or lose most of your marks.

Many candidates, however, make some common mistakes in the exam. Knowing these you can avoid making them further. The common mistakes are:

  • Having lots of ideas.
  • Having undeveloped ideas.
  • Having no/poor explanations or examples.

You need to write around 250-300 words. If you conceive too many ideas, you won’t have enough time to develop the ideas fully. Consequently, you will lose marks. So, don’t make the same mistakes as others do.

Structure of a Good Main Body Paragraph

The structure of a good main body paragraph contains three key elements. They are:

  • Topic sentence
  • Explanation sentences

Please note that although this is a standard model, the structure can change according to the question types.

Topic sentence: It plays a pivotal role in main body paragraphs. In simple words, the topic sentence introduces the key idea, acting as a signpost pointing to what the examiner is going to read.

Explanation sentence: The idea topic sentence states should be explained clearly. Put simply, you have to clearly explain what your topic sentence means. In fact, you answer the question through explanation sentences. The explanation should be 2-4 sentences.

Useful language for explanation: In other words…… That is to say…….. This is because…… The reason is…….. As a result…………. Therefore……………

Example: examples are also crucial for an essay. You need to support your explanation with good and relevant examples. Examples prove highly useful when they are tangible facts because it causes persuasion and makes the argument tough to refute. Good examples contain references to personal experience, well-known people, cultural traditions, and historical events. Bad examples are overly general references, personal opinions, and assumptions.

Some people believe that criminals should be allowed to get an education and enhance their skills while they serve their sentences in prison.

To what extent do you agree or disagree?

Read the following “introduction” paragraph, paying close attention to the outline of the main points:

It is argued by many that convicted criminals should have access to education programmes so that they can develop their skills while serving their sentences in prison. This essay totally agrees with that statement because it reduces crime rates and can positively change incarcerated individuals .

The outline of the main points (in blue ) declares the topics we will use in our supporting paragraphs:

1- prison education can reduce the crime rate 2- it also positively changes incarcerated people

Let’s write the first main body paragraph now. The first sentence states the topic sentence for this paragraph, which needs to be the reflection of what was stated in the introduction paragraph’s outline of the first key point:

“Prison education programs considerably decrease recidivism.”

As you can see that this topic sentence clearly declares the point initially introduced in the introduction paragraph’s outline of the first key point, this builds a clear link between the essay’s introduction and supporting paragraph.

The second, third and fourth sentences are the discussion sentences:

Most prisoners are released from prisons with educational levels and job skills that are extremely low, and that is why they can earn only meagre incomes once freed. Face with the desperate need to make money but the grim reality of odd jobs, many turn back to crime to survive. However, prison education can equip convicted persons with transferable skills that pave the way for the positive transition when they are released. Therefore, it would reduce prison recidivism.

As can be seen, these sentences clarify the topic sentence explaining that prison education helps prison with transferable skills that open up a wide range of career possibilities, this, in turn, refrain them from committing crime again.

The fifth sentence is an example sentence. The example makes the supporting point hard to counter, and this reinforces the argument of the essay and its ability to convince the examiner of the thesis. The example sentence is:

For instance, a study by Monash University found that ex-offenders who receive some vocational training courses cut recidivism to approximately 40 percent.

As you can see that the example directly supports the topic sentence by bringing up factual information. The study is by a renowned university which makes it seems real. It also shows that the prisoners who enrol in prison education programmes are less likely to return to jail.

When grouped, the sentences of the main body paragraph logically unite in a highly persuasive manner:

Prison education programmes considerably decrease recidivism. Most prisoners are released from prisons with educational levels and job skills that are extremely low, and that is why they can earn only meagre incomes once freed. Face with the desperate need to make money but the grim reality of odd jobs, many turn back to crime to survive. However, prison education can equip convicted persons with transferable skills that pave the way for the positive transition when they are released. Therefore, it would reduce prison recidivism. For instance, a study by Monash University found that ex-offenders who receive some vocational training courses cut recidivism to approximately 40 percent. (106 words)

As you can notice that the topic sentence extends upon what was stated in the introduction paragraph’s outline statement, an obvious instance of cohesion at the essay level. Likewise, the argument advances through the discussion and use of a tangible example, and this makes it difficult for the examiner to rebut.

Now look at the second body paragraph and notice how it is written:

Another reason why incarcerated individuals should get an education is that it transforms incarcerated people. In simple words, education in prison changes offenders because it revives humanity, boosts confidence and self-esteem, develops literacy levels, equips them with essential skills and transforms criminals into law-abiding and productive citizens on release. A case in point is Carlos Rosato, who was arrested for armed robbery and sent to prison in New York state for 16 years. He enrolled in an education program of the Bard Prison Initiative, and he earned an Associate degree and a Bachelor degree. Today Carlos Rosato is an engineer and makes $90,000 a year. He is a solid member of his community and is, in fact, a taxpayer. (119 words)

Both the main body paragraphs of the essay are now complete. When joined to the introduction, the composition reads:

It is argued by many that convicted criminals should have access to education programmes so that they can develop their skills while serving their sentences in prison. This essay totally agrees with that statement because it reduces crime rates and can change incarcerated individuals.

Prison education programmes considerably decrease recidivism . Most prisoners are released from prisons with educational levels and job skills that are extremely low, and that is why they can earn only meagre incomes once freed. Face with the desperate need to make money but the grim reality of odd jobs, many turn back to crime to survive. However , prison education can equip convicted persons with transferable skills that pave the way for the positive transition when they are released. Therefore , it would reduce prison recidivism. For instance , a study by Monash University found that ex-offenders who receive some vocational training courses cut recidivism to approximately 40 percent.

Another reason why incarcerated individuals should get an education is that it transforms incarcerated people . In simple words , education in prison changes offenders because it revives humanity, boosts confidence and self-esteem, develops literacy levels, equips them with essential skills and transforms criminals into law-abiding and productive citizens on release. A case in point is Carlos Rosato, who was arrested for armed robbery and sent to prison in New York state for 16 years. He enrolled in an education program of the Bard Prison Initiative, and he earned an Associate degree and a Bachelor degree. Today Carlos Rosato is an engineer and makes $90,000 a year. He is a solid member of his community and is, in fact, a taxpayer.

Please notice the way the sentences and paragraphs use linking devices to connect themselves together and how to build the overarching argument of the essay. Cohesion at the sentence level is underlined . Cohesion at the essay level is highlighted in blue .

Step 5 – Write The Conclusion:

The conclusion paragraph reiterates the writer’s main ideas and closes the essay. It’s far easier than the introduction and main body paragraphs because it contains the information that has already been imparted earlier in the response. Alternatively, you can propose a solution to a problem or issue or offer something that would be widely accepted regarding the topic discussed in your essay.

Let’s start with the common mistakes. Many candidates often make some mistakes. So, you should not:

  • Introduce new ideas.
  • Try to be entertaining.
  • Be too general.
  • Repeat exactly the same thing as in the rest of your essay.
  • Use the wrong cohesive devices.

Many candidates make the mistake of introducing new ideas in their “conclusion”. Bear in mind that this is a big mistake. New ideas shouldn’t be produced in your “conclusion” at all. All you need do is to state the ideas you have already discussed in the previous paragraphs or offer a solution to a problem that would be widely accepted. If you outline any new idea in your conclusion, you’ll get a lower mark because the conclusion is just paraphrasing or summarizing of what you’ve already said, or offering a universally accepted solution – nothing else.

Many candidates also try to finish their essays by being entertaining or interesting. There are no marks for being entertaining or interesting in your conclusion! There are only marks for writing an accurate conclusion! So, don’t try to write an entertaining conclusion.

Being too vague is another error while writing a conclusion. You need to be as specific as possible like the rest of your essay. The more specific you are, the higher the chance you get a higher band score. So, don’t be too general.

Some candidates repeat the same thing as in the rest of the essay. You need to paraphrase; you need to write the same thing but in a different way.

Finally, many people use wrong cohesive devices which ultimately negatively affect their scores. Therefore, don’t use the wrong cohesive devices.

Let’s look at some inappropriate cohesive devices:

All in all – it is a very inappropriate way to begin your conclusion. This is because it’s very informal. You need to be academic as the IELTS essay is academic in nature. You need to be as formal as possible. So, don’t use “all in all”. In sum – it means just summarizing things. In the conclusion paragraph, you are not just summarizing your main ideas but also giving your opinion. So, the phrase ‘in sum/in summation’ is not quite appropriate for that reason.

To sum up – it is the same as ‘in sum’. So, you shouldn’t use it either.

Finally – it’s saying that you’ve got a final point and it would imply that you’ve got a new point. As we said before, you shouldn’t have a new idea in your conclusion. So, don’t use the word ‘finally’.

In a nutshell – like the phrase “all in all”, this is also informal. So, don’t use it.

Now let’s look at the cohesive devices you should use in your conclusion. They are: ✓ To conclusion ✓ In conclusion

These are two simple phrases you can use at the beginning of your “conclusion” paragraph.

Structure of a Good Conclusion Paragraph: The structure of a good conclusion consists of two essential things. They are: a) Summary of main points b) Opinion

Please reread the introduction and main body paragraphs written above prior to preparing for the conclusion paragraph. Once you read it, let’s continue to the techniques and examples of writing the “conclusion” part.

As a recommended structure, the first sentence of the conclusion paragraph should summarize the topics discussed in the main body paragraphs. The topics are:

→ Prison education programs considerably decrease recidivism. → it transforms incarcerated people.

Grouped into a single sentence, these two ideas would read:

In conclusion, education in prison causes considerable recidivism reduction and also changes convicted persons in a positive way.

The second sentence has to rephrase the opinion of the introduction paragraph in a different manner. Here is the original outline sentence:

This essay totally agrees with that statement…

Thus, it is clear that incarcerated men and women should get an education while in prison.

Now the conclusion paragraph is finished. Notice how its sentences connect with one another and with earlier parts of the essay:

In conclusion, education in prison causes considerable recidivism reduction and also changes convicted persons in a positive way. Thus, it is clear that incarcerated men and women should get an education while in prison.

Please note that, if you wish to add one or two lines of suggestion or proposition that is related to the topic and offers something widely accepted, do that at the end of the conclusion.

Example of such a suggestion or proposition –

“It is expected that prison authority would take measures to educate inmates and train them so that they can become responsible members once they complete their prison sentence and get back to normal life.”

Essay Topic:

Essay Answer: I t is argued by many that criminals should have access to education programmes so that they can develop their skills while serving their sentences in prison. This essay totally agrees with that statement because it reduces crime rates and can change incarcerated individuals.

Prison education programmes considerably decrease recidivism. Most prisoners are released from prisons with educational levels and job skills that are extremely low, and that is why they can earn only meagre incomes once freed. Face with the desperate need to make money but the grim reality of odd jobs, many turn back to crime to survive. However, prison education can equip convicted persons with transferable skills that pave the way for the positive transition when they are released. Therefore, it would reduce prison recidivism. For instance, a study by Monash University found that ex-offenders who receive some vocational training courses cut recidivism to approximately 40 percent.

Another reason why incarcerated individuals should get an education is that it transforms incarcerated people. In simple words, education in prison changes offenders because it revives humanity, boosts confidence and self-esteem, develops literacy levels, equips them with essential skills and transforms criminals into law-abiding and productive citizens on release. A case in point is Carlos Rosato, who was arrested for armed robbery and sent to prison in New York state for 16 years. He enrolled in an education program of the Bard Prison Initiative, and he earned an Associate degree and a Bachelor degree. Today Carlos Rosato is an engineer and makes $90,000 a year. He is a solid member of his community and is, in fact, a taxpayer.

(Approximately 300 words)

Learning to write an essay at a band 9 level takes a lot of practice. Use this article to acquire the technique, then do practice as much as possible on all the five types of questions. Best of luck!

Leave a Reply Cancel reply

Your email address will not be published. Required fields are marked *

Privacy Overview

  • Skip to primary navigation
  • Skip to main content
  • Skip to primary sidebar
  • Skip to footer

IELTS Advantage

IELTS Advantage

IELTS Preparation Courses

Band 9 Student Answer With Examiner’s Report

My essay correction service has been running for just over a month now and I was delighted to mark my first Band 9 essay yesterday.

This is truly a remarkable achievement from the student in question. I spoke to an IELTS examiner recently who said he has only awarded a Band 9 three times in an 8 year career.

It also demonstrates that if you can pinpoint your weaknesses and work hard to fix them, then very high IELTS scores are possible.

Below is his essay with my comments in red, then my report and finally a sample essay. Please note that this report is much shorter than normal because there were no recommendations for improvement. To see what a report for a lower level essay looks like please visit our essay correction service .

Question- Many museums charge for admission while others are free. Do you think the advantages of charging people for admission to museums outweigh the disadvantages?

Some museums have an admission charge while some do not. In my opinion, the drawbacks of an entrance fee are eclipsed by its benefits in the sense that the income will be ploughed back into operation and development of the museums.

Excellent. You have outlined what you are going to talk about and at the same time made your opinion very clear. This is exactly what you need to do to make it clear to the examiner what you think about the question and also helps them to follow the rest of your essay.

A major disadvantage of an admission fee is the possibility of reducing the number of visitors. Museums house exhibitions and artefacts of great educational and historical value. If the chief aim of a museum is to introduce the local community, admission should be free to the public and visitors. Take some folk museums in Hong Kong, which preserve historic relics and display folk customs, for example. Admission to these folk museums, which are often monuments, is free of charge. If they had charged an entrance fee, many might have turned to other activities.

Excellent. Clear and relevant main reasons and fully developed idea. It was great to see how you took one central idea and developed it fully with explanations and a specific example.

Granted, an entrance fee might have a negative effect on the admission figures, but an income is favorable to museums in terms of operation. Museums feature educational exhibitions at times, and this could not have been done without a sum of money expended on hiring professionals and buying equipment. The Hong Kong Space Museum, for instance, has monthly exhibitions on different issues and professional docents are employed to take visitors on a guided tour around the museum. This example speaks volumes about how a reasonable admission charge is advantageous to the operation of a museum.

As above. I particularly like the way you linked both main ideas in the first sentence.

In conclusion, the disadvantages of an admission fee are overshadowed by the benefits accruing from a stable source of income. Therefore, having weighed up the pros and cons, I am convinced that museums should charge an entrance fee for the sake of operation and development.

Very elegant conclusion. You have summarised your main points and reiterated your opinion. Well done.

Task Response- Band 9

Your answer fully addresses all parts of the task.

Your answer presents a fully developed position in answer to the question with relevant, fully extended and well supported ideas.

It is very clear that you think the benefits outweigh the drawbacks and the whole of your essay supports this.

Coherence and Cohesion- Band 9

This was the most impressive part of your essay. I really liked your introduction and conclusion in particular. They really helped to bring everything together and hammer your point across.

I also really liked the way you linked your ideas together. The whole essay felt like a really tight, cohesive piece of writing.

You have used cohesion in a way that attracts no attention.

Your paragraphing is excellent.

Very rarely do I have to read an IELTS essay only once to understand everything. That is the sign of a truly great essay.

Vocabulary- Band 9

You naturally use less-common words to convey very precise meaning and there are no mistakes at all.

Grammar- Band 9

You use a wide range of appropriate structures and there are no mistakes.

Overall- Band 9

This is the first time a student has submitted a Band 9 essay to my service and I am thrilled that you have reached this level. I am very happy I was able to help you reach this goal and I hope you can continue at this level in the future.

Below is a sample answer. I have taken a different approach just to show you a different way of answering the same question.

Sample Answer

Lots of museums charge a fee while others do not. This essay thinks that the benefits of charging do not outweigh the drawbacks because open access to relics and art is more important than generating money that the government should supply in any event.

The main disadvantage is that high fees exclude a large proportion of the population, especially in less developed countries. Many people in poorer countries have just enough money for food and shelter. Exhibits are one of the few cultural activities they can enjoy free of charge. For example, Egypt has millions of people living in poverty, but also a rich and ancient culture and it is therefore important that everyone gets to experience these artefacts. Another big negative is that students and children who are learning about the world may not be able to visit. It would be a huge shame if art students could not see their favourite painters or sculptor’s work in real life because their finances could not cover the cost.

Despite this, there are some who say that museums are unsustainable without the money they might get from ticket sales. They say that this allows the building to remain open and it is better that some people get to experience it, rather than none at all. To this I would say that the government should step in and cover the cost because culture is as important as anything else it spends money on. For instance, in the UK there have been huge government spending cuts over the last few years, but the museums have not had their funding reduced because of their importance to the country’s cultural heritage.

In conclusion, although some might say that places of culture should be run like a business, the cost to the education and heritage of the country is too great and they should remain free to all.

For more sample essays and Task 2 lessons, visit out Task 2 page .

If you have any questions or comments, join the conversation over on our Facebook page .

ielts essay on art band 9

About Christopher Pell

My name is Christopher Pell and I'm the Managing Director of IELTS Advantage.

I started IELTS Advantage as a simple blog to help 16 students in my class. Several years later, I am very humbled that my VIP Course has been able to help thousands of people around the world to score a Band 7+ in their IELTS tests.

If you need my help with your IELTS preparation, you can send me an email using the contact us page.

IELTS Writing Samples Band 9

Having more money and less free time is better than earning less money and having more free time. discuss both views and state your opinion., some people think that the amount of money spent on library is a waste as a new technologies are developing and replacing library functions. do you agree or disagree, some children spend hours every day on their smartphones. why is this the case do you think this is a positive or a negative development, many university students want to learn about different subjects in addition to their main subjects. others feel it is more important to give all their time and attention to studying for their qualification., some people say that the main environmental problem of our times is the loss of particular species of plants and animals. others say that there are more important environmental problems. discuss both these views and give your opinion., some university students want to learn about other subjects in addition to their main subjects. others believe it is more important to give all their time and attention to studying for a qualification. discuss both these views and give your opinion. give reasons for your answer and include any relevant example for your own knowledge or experience., in many countries today, women have full-time jobs. therefore, it is reasonable to share housework equally between men and women. to what extent do you agree or disagree with this statement give reasons for your answer and include any relevant examples from your own knowledge or experience. write at least 250 words., in many countries, people are now living longer than ever before. some people say an ageing population creates problems for governments. other people think there are benefits if society has more elderly people., the most importan aim of science is to should be to improve people's life. to what extent do you agree or disagree with this estatement., some people believe that increasing tax on various industries will reduce pollution whereas others believe that there are better ways. discuss both views and give your opinion., some people believe that unpaid community service should be compulsory in high school programmes (for example, working for a charity, improving the neighbourhood or teaching sports to younger children). do you agree or disagree, some people think that paying taxes is a big enough contribution to society, while others think people have more responsibilities as a member of society than only paying taxes. discuss both views and give your opinion., in many countries around the world, rural people are moving to cities, so the population in the countryside is decreasing. do you think this is a positive or a negative development, some children spend hours every day on their smartphones. why is this the case do you think this is a positive or a negative development, some people say that modern innovation brings about more problems than benefits do you agree or disagree, some people believe that teenagers should concentrate on all school subject. but others believe that teenagers should focus on the subject that they are good at or they find the most interest. discuss both sides and give your opinion., some people say it is a waste of time to plan for future. it is more important to focus on present. do you agree or disagree, do you agree or disagree with the following statement it is more important for students to understand ideas and concepts than it is for them to learn facts.use specific reasons and examples to support your answer, 2.it is becoming increasingly popular to have a year off between finishing school and going university. what are the advantages and disadvantages of this trend, modern children are suffering from the diseases that were once considered to only be meant for adults. obesity is a major disease prevalent among children. what are its causes, and what solutions can be offered.

  • Unlimited Essay Checks: Practice and perfect your skills.
  • Detailed Error Analysis: Spot every mistake.
  • In-Built Grammar Checker: Say no to grammatical errors.
  • Personalized Suggestions: Know how to boost your score.
  • Progress Tracking: View your checked essay history.
  • Still thinking? We have a 14-day money-back guarantee. Take a leap of faith!

VIDEO

  1. How to Structure a Band 9 Opinion Essay

  2. Ielts List of Headings || IELTS Reading || ielts reading list of headings

  3. IELTS Writing Task 1 Line Graph

  4. IELTS ESSAY 2024. ANSWER BAND 9

  5. Ielts Band 9 Essay Sample

  6. Generate Band 9 IELTS Ideas After Doing This

COMMENTS

  1. Band 9 sample essay about art

    Home » IELTS Writing Task 2 » 35 Sample Band 9 IELTS Essays » Band 9 sample essay about art Many governments in the world spend large amounts of money on art, which helps to improve the quality of people's lives. However, governments should spend money on other things rather than art.

  2. Band 9 Essay Sample

    Band 9 IELTS essay sample. Art has always played a vital role in enriching cultures all over the world. However, people are showing less and less interest in art these days. In this essay, I will analyse the reasons behind this trend and also suggest possible ways to boost people's interest in arts. While it is true that art enriches culture ...

  3. 35 Sample Band 9 IELTS Essays

    35 Sample Band 9 IELTS Essays. Take a look at these 35 sample Band 9 IELTS essays for writing task 2 of the IELTS exam. Task 2 can cover a wide range of essay topics for the IELTS writing task section of the test, so preparation is key. Use the following samples when preparing your IELTS essays to see how close you are to a band 9!

  4. Band 9 IELTS Essay Sample

    Band 9 IELTS essay sample. Whether art and artists should receive government funding has always been a topic of debate. Some people argue that the government should spend its money on more important things like health and education instead of sponsoring artists and art events. I do not quite agree with this view.

  5. PDF IELTS Band 9 Sample Essays

    IELTS Band 9 Sample Essays Sample Essay #1 - Two Part Question In some countries, the number of people visiting artgalleries is ... In certain locations around the world, the numberof people visiting art galleries is declining. This essay shall outline someof the reasons for this trend and then go on to suggest ways in which thisissue could ...

  6. Band 9 IELTS Essay Sample

    Band 9 IELTS essay sample. According to some people, museums should only display local and national works of art while others insist that they should showcase artifacts from all over the world. In my opinion, museums are places to preserve and showcase art and equal importance should be given to artworks from all over the world.

  7. IELTS Writing Task 2 : Art (Opinion essay)

    Task 02: Art Some people say that art (e.g. painting, music, poetry) can be made by everyone whereas others believe that it can be only made by those with special abilities. Discuss both views and give your opinion. Sample Answer: While some people believe that everyone has the ability to create art, others believe that […]

  8. Band 9 Essay: Creative artist should always…

    Band 9 Essay: Creative artist should always be given the freedom to express their own ideas in words, pictures, music or film in whichever way they wish. There should be no government restrictions on what they do. To what extent do you agree or disagree with this option Art and creativity impact our society. Individuals

  9. IELTS Writing task 2: Art questions and answer samples

    Here you can find common IELTS essay questions for "Art" topic. #1 . For a long time art has been considered an essential part of all cultures in the world. However, nowadays people's values have changed, and we tend to consider science, technology and business more important than arts.

  10. IELTS Band 9 Writing Samples: Task 2 Essays

    IELTS Band 9 Writing Samples: Task 2 Essays. Tim James. June 19, 2021. One of the best ways of learning how to write better is to simply read sample IELTS band 9 essay answers, and that is exactly what we have here: 10, Band 9 sample IELTS essays. Each essay is followed by a teaching point to show you why it is a band 9 IELTS essay.

  11. IELTS Writing Task 2 Problem/Solution Essay Band 9 Model Answer-Art

    Here is an example showing how to write an IELTS problem/solution essay. There are two logical structures for the body paragraphs. Option 1) Body Paragraph 1=Problem 1 and Problem 2 Body Paragraph 2=Solution 1 and Solution 2. Option 2) Body Paragraph 1=Problem 1 and Solution 1 Body Paragraph 2=Problem 2 and Solution 2.

  12. Art and music are considered some of the fundamental..(Task 2 Band 9)

    This is a band 9 IELTS essay sample, written by an actual IELTS examiner. You should read this 9 band sample essay to learn how to structure a high band IELTS Writing. ... This essay posits that the integration of art and music into the educational curriculum is imperative, not only for cultivating a well-rounded intellect but also for ensuring ...

  13. Band 9 IELTS Essays

    Band 9 IELTS Essays. Here at ielts-practice.org we have a huge collection of band 9 IELTS essay samples. Click on the links below to read our band 9 essay samples. IELTS essay topics tend to repeat. It is, therefore, imperative that you practice writing essays on topics asked in recent IELTS exams. We are adding more essays to this page, so ...

  14. IELTS Writing Task 2: Band 9 Sample Essay

    IELTS Band 9 sample essay. Band 9 Sample answers are useful as study guides for IELTS preparation for the IELTS Writing Task 2 essay - especially for a band 9 IELTS essay. Having access to previously completed work that you can have confidence in will show you what you are missing! Take a look at these sample task 2 essay questions to help ...

  15. IELTS Band 9 Essays: How to Write 9 Band Essays in IELTS

    IELTS writing task involves two parts, task 1 and task 2. In task 1, the candidate has to write a summary of a given set of data or diagrams. In task 2, the candidate has to write an essay on a given scenario or problem in above 250 words. This is a crucial part of the writing test and carries 66 per cent marks out of the total writing score.

  16. 7 IELTS Essay Samples of Band 9 Students

    Finally, here are the 7 examples of Band 9 essays. IELTS Writing Sample Essay 1 -Fresh water demand causes and measures. IELTS Writing Sample Essay 2 - Forests are the lungs of the earth. IELTS Writing Sample Essay 3 - Job and money. IELTS Writing Sample Essay 4 - Aim of University Education.

  17. IELTS Band 9 Essays

    An IELTS Band 9 Essay is one that shows the examiner that you are an expert user of English. The official IELTS scale describes an expert user in the following way: "The test taker has fully operational command of the language. Their use of English is appropriate, accurate and fluent, and shows complete understanding.".

  18. 100 Band 7, 8 + 9 IELTS Writing Task 2 Essay Samples

    In this blog post, we have compiled a list of 100 Band 7, 8, and 9 IELTS Writing Task 2 essay samples to help you improve your writing skills and boost your chances of achieving a high score on the exam. These sample essays cover a wide range of topics, from education and technology to health and environment, and are a valuable resource for ...

  19. IELTS Essay Samples of Band 9

    Schools should teach their students how to survive financially in the world today (agree/disagree) - Sample essay 1. Some people believe that teaching music in schools is vital, while others think it is unnecessary (opinion) - Sample essay 2. Teachers should be required to conform to a dress code (agree/disagree) - Sample essay 3.

  20. Analysing a Band 9 Sample Answer for IELTS Writing Task 2

    Sample Band 9 Answer. Here's my answer to the above question: As an increasing number of people go to university, it is now common that graduates cannot find employment. This is a worrying situation, but there are some solutions to it. The current problem of graduate unemployment is largely caused by there being too many graduates.

  21. How to write an IELTS Essay at band 9 level

    Step 2 - Plan The Structure: Candidates who get a higher mark in IELTS writing task 2 always plan their arguments and ideas. A good plan helps them organise their ideas, and then structure their essay before they write it, saving their time and helping them to write a well-structured, coherent essay.

  22. Band 9 Student Answer With Examiner's Report

    Task Response- Band 9. Your answer fully addresses all parts of the task. Your answer presents a fully developed position in answer to the question with relevant, fully extended and well supported ideas. It is very clear that you think the benefits outweigh the drawbacks and the whole of your essay supports this.

  23. IELTS Writing Samples Band 9

    9. band. The most importan aim of science is to should be to improve people's life. To what extent do you agree or disagree with this estatement. Owing to the rapid growth of research and innovation in science, it is believed that the aim of science is to ameliorate living standards in society.

  24. SAVE THIS ESSAY FOR 8 BAND OR ANY Query PLEASE CONTACT 9872499119 #

    6 likes, 0 comments - marigold_ielts on February 9, 2024: "SAVE THIS ESSAY FOR 8 BAND OR ANY Query PLEASE CONTACT 9872499119 #ielts #reels #reelsinstagram #training #essay"